Download as pdf or txt
Download as pdf or txt
You are on page 1of 131

PREPARED BY MOSES KAZEVU JR

SURGERY EXAMINATION PREPARATION


SECTION 1: TRUE/FALSE
1. Clinical diagnosis of hypovolemic shock may be made with the following:
A. Symptoms will include feeling dizzy, faint, nauseated or very thirsty-True
B. Cardiovascular signs such as tachypnea, hypoventilation-False
C. The absence of perfusion may be assessed by skin signs such as loss of skin
turgor and capillary refill on nail beds-True
D. Urine output measurements via urinary catheter-True

2. Concerning blood transfusions


A. Modern medical practice commonly uses only components of the blood such
as red blood cells, white blood cells, plasma, clotting factors and platelets-
True
B. Transfusion transmissible infections which are tested on all donated blood
include TB, malaria, typhoid to ensure that the patient is receiving compatible
blood-False
C. All donated blood should also be tested for the ABO blood group and Rh
blood group system to ensure that the patient is receiving safe and non-
infectious blood- True**
D. Symptoms of acute hemolytic reactions due to a mismatch in ABO blood type
between the donor and the recipient include fever, chills, chest pain, back pain,
increased heart rate, shortness of breath and hypotension within 24 hours of
transfusion-True

3. Glasgow coma scale


A. Is a neurological scale which aims to give an objective way of recording the
conscious state of a person- True
B. Is composed of eye, verbal and motor responses- True
C. Glasgow coma scale is high as ≥13 signifies minor head injury- True
D. The light response and size of the pupil are part of the Glasgow coma scale
assessment-False
PREPARED BY MOSES KAZEVU JR

4. Symptoms of thyroid cancer include the following


A. A nodule in the thyroid region of the neck-True
B. Positive Kocher’s test-True
C. Pain in the anterior region of the neck and changes in voice-True
D. Virchow’s node (Troisier’s sign)-True

5. Thyroid carcinoma
A. Frequency- Papillary, Follicular, medullary and anaplastic thyroid cancer-
True
B. Frequency- Medullary, papillary, anaplastic and follicular thyroid cancer-
False
C. All anaplastic thyroid cancers are considered to have poor prognosis-True
D. Anaplasia is a condition of cells with poor cellular differentiation-True

6. Sentinel node biopsy


A. A type of biopsy in which you locate and remove the lymph nodes that the
primary tumor is most likely to spread to-True
B. The dissecting out and removing all the lymph nodes that provide lymphatic
drainage for cancerous areas in an effort to prevent lymphatic spread of cancer
cells-False
C. Surgical procedure used to determine whether cancer has spread beyond a
primary tumor into your lymphatic system-True
D. It is used most commonly in evaluating breast cancer and melanoma-True

7. Osteomyelitis
A. Most of the infections are caused by Staphylococcus aureus while in patients
with sickle cell disease, the most common causative agent is salmonella- True
B. Sequestrum is a layer of new bone growth resulting from the stripping-off of
the periosteum by the accumulation of pus within the bone and new bone
growing from the periosteum-False
C. Acute osteomyelitis almost invariably occurs in children because of rich blood
supply to the growing bone while adults are affected, it is because of
compromised host resistance-True
D. Involucrum is piece of dead bone that has become separated during the
process of necrosis from normal bone-False
PREPARED BY MOSES KAZEVU JR

8. Causes of obstructive uropathy include


A. Posterior urethral valve, phimosis, hypospadias, hydrocolpos-False**
B. Balanitis, schistosomiasis-True**
C. Benign Prostatic hypertrophic, urolithiasis-True
D. Glomerulonephritis, pyelonephritis-False

9. Femoral hernia
A. Can be controlled by a truss-False
B. Of all hernias it is the least likely to become strangulated-False
C. Operative procedures include Lockwood and McEvedy procedures-True
D. Differential diagnosis may be psoas abscess, saphena varix-True

10.Symptoms of lower urinary tract obstruction


A. Weak stream, hesitancy- True
B. Dribbling, straining to void- True
C. Polyuria, oliguria-False
D. Frequency, urgency, urge incontinence- True

11.Which are the causes of dysphagia


A. Carcinoma, goiter, thymoma, mediastinal lymphadenopathy, aortic
aneurysm- True
B. Motor neuron disease, rabies, tetanus, pseudobulbar paralysis-True
C. Achalasia- True
D. Pneumonia, TB of the spine-False

12.Differential diagnosis of a scrotal swelling


A. Collection of fluid in the tunica vaginalis-True
B. Femoral hernia-False
C. Hematocele-True
D. Seminoma-True
PREPARED BY MOSES KAZEVU JR

13.TB spine
A. The bacilli spread from the lung to the intervertebral disc space via the
lymphatics-False
B. Infection weakens the vertebrae causing collapse under the weight of the body
causing gibbus deformity- True
C. Signs and symptoms may include spastic paralysis-True
D. The standard recommended tuberculin test is the Mantoux test will be
negative to confirm the diagnosis-False

14.Troizier’s sign is
A. Left supraclavicular lymphadenopathy- True
B. Virchow’s lymphadenopathy- True
C. Due to metastasis from thoracic or abdominal malignancy- True
D. Present in portal hypertension- False

15.The following changes occur in stored blood


A. Increased [K+]- True
B. Reduced pH- True
C. Reduced viability of platelets and leucocytes-True
D. Increased coagulation factors-False**

16.In terms of classification of hemorrhagic shock, the following are the


characteristic of class II
A. Loss of 750ml of blood volume- True**
B. Loss of 15-30% of blood volume- True
C. The patient’s systolic and diastolic falls-False
D. Heart rate of 100-120 beats/minute- True

17.Principles of treatment in empyema thoracis (ET)


A. Drain pus-True
B. Sterilize pleural cavity-True
C. Encourage lung expansion-True
D. Prevent complication-True
PREPARED BY MOSES KAZEVU JR

18.Empyema thoracis (ET)


A. Is pus in mediastinum-False
B. Is pus in mediastinum and pleural cavity-False
C. Is pus in the lung-False
D. Is pus in the pleural cavity-True

19.Measures to be taken when managing shock include:


A. Use of inotropes and vasoconstrictors- True
B. Administration of oxygen- True
C. Uncross matched O negative pack cell may be used-True**
D. Position and rest-True

20.Complications of underwater seal intercostal drain insertion include:


A. Pneumothorax- True
B. Injury to neurovascular bundle-True
C. Lung and liver trauma-True
D. Surgical emphysema-True

21.An indirect inguinal hernia would lie in relation to the inferior epigastric artery
A. Medial-False
B. Lateral-True
C. Anterior-False
D. Posterior-False

22.Fissue-in-ano is located characteristically


A. Midline anteriorly-False
B. Midline posteriorly-True
C. Right lateral quadrant-False
D. Left lateral quadrant-False

23.Cancer which does not frequently metastasis to bone


A. Breast-False
B. Rectum-True**
C. Prostate-False
D. Thyroid- False
PREPARED BY MOSES KAZEVU JR

24.Obstructive jaundice
A. Is associated with biliary atresia-True
B. Is associated with itching of the skin-True
C. A patient with obstructive jaundice passes pale stool-True
D. Obstructive jaundice can be diagnosed by supine X-ray-False

25.When determining the depth of a burn


A. Knowledge of the type of injury is important- True
B. The presence of blisters is of no clinical significance- False
C. Impairment of sensibility of the burned area should be tested- True
D. The presence of severe pain denotes a full thickness burn- False

26.In thyroid function


A. The stage of coupling in thyroid hormone synthesis is catalyzed by deiodinase
enzyme- False
B. T4 is metabolically more active than T3- False
C. Thyroid hormone lack in children may affect physical growth and mental
development- True
D. TSH, T3, T4 radioisotope thyroid scan are useful indices for thyroid gland
activity-True

27.Regarding iodine metabolism


A. Uptake is solely by thyroid gland tissue-False
B. Daily iodine requirement is 100 micrograms- False**
C. Iodine is trapped as iodine-False (it is trapped as iodide)
D. Iodine is organified by carboxylase enzyme-False

28.The parafollicular cells of thyroid gland secrete


A. Calcitonin- True
B. Amyloid-False
C. Parathormone- False
D. Follicle stimulating hormone- False
PREPARED BY MOSES KAZEVU JR

29.In an amputated limb the following would account for pain in the stump
A. Phantom limb- True
B. Infected or ulcerated stump- True
C. Sequestrum- True**
D. Neuroma-True

30.Spinal injury causing quadriplegia


A. Fracture dislocation involving C5-C6 is likely cause-True
B. Fracture dislocation involving T6-T7 is likely cause-False
C. Fracture dislocation involving C6-C7 is likely cause-True
D. Fracture dislocation involving T12-L2 is likely cause-False

31.Acute abdomen can be caused by


A. Perforation duodenal ulcer-True
B. Sigmoid volvulus- True
C. Pancreatitis-True
D. Subcutaneous lipoma-False

32.Pathological fractures may be associated with


A. Scurvy-True
B. Hyperparathyroidism-True
C. Bone cysts-True
D. All of the above-True

33.In a jaundiced patient with palpable gallbladder, the cause of jaundice is most
likely to be
A. Gallstones in the gall bladder-False
B. Gallstones in the common bile duct-False
C. Cancer of head of pancreas- True
D. Cancer left kidney-False

34.Hashimoto’s disease
A. Is common in post-menopausal women-True**
B. On inspection one lobe of the thyroid is larger than the other-False
C. Symptoms of hypothyroidism are absent-False
D. Blood test may show antibodies to the thyroid gland-True
PREPARED BY MOSES KAZEVU JR

35.Burns:
A. History, taking should always include- Time of burn, cause, smoke inhalation,
trauma- True
B. On examination the severity is determined by burn depth, burn size, areas of
circumferential full thickness-True
C. Full thickness burns will produce very painful charred skin- False
D. Escharotomy may help in reducing respiratory embarrassment- True

36.Fibroadenoma
A. Commonest benign breast tumor- True
B. Intracanalicular and pericanalicular are of same consistency- False
C. Excision provides permanent cure-True
D. FNAC is recommended before surgery- False**

37.Phyllodes tumor on histology


A. Might be benign- True
B. Might be borderline- True
C. Might be frankly malignant- True
D. Cellular stroma is characteristic-True

38.Breast tissue lies


A. Within retromandibular space-False
B. Deep to pectoral fascia-False
C. Within superficial fascia- True
D. Only “B” correct- False

39.Fibrocystic breast change presents as


A. Fibroadenosis-True
B. Cystic formation-True
C. Epitheliosis-True
D. Fibrosis and lymphocytosis-True

40.A 3/0 suture is


A. Larger in diameter than 4/0-True
B. Small in diameter than 4/0-False
C. Is too small for skin closure-False
D. Non-above correct-False
PREPARED BY MOSES KAZEVU JR

41.Needle holders come in several varieties:


A. Those with jaws with teeth- True
B. Those used for injection-False
C. Those with teeth-True
D. All above correct-False

42.In burns
A. A man with 30% superficial burns does not require fluid therapy-False
B. A female patient with 20% superficial burns does not require ATT on
admission-False
C. Parkland’s formula considers fluid therapy in the first 36 hours-False
D. Browder and Lund Charts are used to estimate percentage surface burnt-True

43.A diagnosis of an ulcer can be made by examining


A. The facial appearance of the patient-False
B. The edges of an ulcer-True
C. Regional lymph node involving an ulcer-True
D. Looking at its location on the body-True

44.Intestinal obstruction
A. Can be classified depending on the age of the patient- True
B. Can be made on erect clinical examination- False
C. Can be due to electrolyte imbalance- True
D. Pyloric stenosis is caused by lower intestinal obstruction-False

45.Hyper-resonant chest is likely to be caused by


A. Hemothorax- False
B. Pleural effusion- False
C. Pneumothorax- True
D. Tension pneumothorax-True

46.Paradoxical (diaphragmatic) respiration can be caused by


A. Odontoid fracture dislocation-False
B. C2/C3 hangman’s fracture-False**
C. C5/6 fracture dislocation-True
D. T12/L1 fracture-False
PREPARED BY MOSES KAZEVU JR

47.Examination of localized swelling


A. Should include determining its anatomical plane- True
B. Should include determining its size-True
C. It is unnecessary to determine the consistence of the swelling-False
D. Slipping sign is helpful in diagnosing lipoma-True

48.Intravenous solutions available


A. Normal saline and ringers lactate are isotonic solutions-True
B. 8% normal saline and 10% dextrose are hypertonic solutions-True
C. 10% dextrose is very effective in correcting hypotensive crises- False
D. All correct except (b)-False

49.In wound healing


A. Cytokines are solely produced by hemopoietic cells- False
B. Cytokines optimize cellular response in inflammation-True
C. Interleukins are not classified as cytokines-False
D. Macrophages actively synthesize cytokine growth factors for angiogenesis-
True

50.Factors that delay wound healing include


A. Poor blood supply for nutrition support-True
B. Infection-True
C. Poorly controlled diabetes mellitus-True
D. All above except (c)-False

51.Excessive epithelialization
A. Is related to lack of contact inhibition-True
B. May result in keloids-True
C. May result in hypertrophic scars-True
D. All of the above-True
Sodium= 135-145mEq/l
52.Wound contraction Potassium= 3.5-
A. May result in contractures and strictures-True 4.5mEq/l
B. A smaller scar-True
Chloride= 96-106mEq/l
C. Is effected by myofibroblasts-True
D. All above are correct- True Calcium=9-10mg/dl
PREPARED BY MOSES KAZEVU JR

53.Inguinal hernia
A. Indirect hernia is more common than direct hernia- True
B. Strangulation is more common with direct inguinal hernia than with indirect
hernia-False
C. Obstruction is more common with indirect inguinal hernia than direct hernia-
True
D. Direct hernia is likely to protrude through Hesselbachs triangle- True

54.Differential diagnosis of inguino-scrotal mass include


A. Hernia-True
B. Hydrocele-True
C. Meningocele-False
D. Lipoma-True

55.Breast cancer
A. Carcinoma in-situ implies tumor has not yet breached basement membrane-
True
B. Lobular carcinoma tends to be multicentric in origin more than ductal
carcinoma-True
C. Ductal carcinoma tends to be multicentric in origin more than lobular-False
D. None of the above-False

56.Unilateral renal artery obstruction may be associated with


A. Increased renin release by the obstructed kidney-True
B. Angiotensin release and increased aldosterone production by adrenal cortex-
True
C. Increased sodium and water-True
D. Hypotension-False

57.Chronic bilharzia can cause


A. Bilateral ureteric obstruction-True
B. Transitional cell carcinoma-False
C. Bilateral hydronephrosis-True
D. Squamous cell carcinoma of the bladder-True
PREPARED BY MOSES KAZEVU JR

58.Acute appendicitis
A. Pain can be referred to the right subcostal region-True*from answer scheme
B. Pain can shift to right iliac fossa-True
C. Can be associated with vomiting-True
D. Can be associated with left hip joint flexion-True

59.Radial nerve
A. Is formed from posterior cord-True
B. Innervates all extensor compartment muscles except anconeus-False
C. Is prone to injury in fractures of shaft of humerus-True
D. Injury results in wrist drop-True

60.In shoulder dislocation


A. Posterior dislocation is commoner than anterior dislocation-False
B. Axillary nerve is at risk of injury-True
C. Kocher’s maneuver may be employed to reduce dislocation-True
D. All above are correct except (c)-False

61.In acute appendicitis


A. Central or epigastric pain radiates to right iliac fossa-False
B. Central or epigastric pain shifts to right iliac fossa-True
C. Pain precedes nausea and vomiting-True
D. Nausea and vomiting precedes pain-False

62.The edge of tuberculous ulcer is


A. Shelving-False
B. Rolled up-False
C. Undermined-True
D. Punched out-False

63.Secondary metastasis to the spine can originate from cancer of


A. Thyroid-False** (some can)
B. Breast-True
C. Lung-True
D. Cancer-True
PREPARED BY MOSES KAZEVU JR

64.Concerning blood groups


A. Blood group A is associated with increased risk of gastric cancer- True
B. The risk of gastric cancer is associated with blood group B-False
C. The risk of gastric cancer is associated with blood group O-False
D. Only (b) and (c) are correct-False

65.Blood products shelf life


A. Whole blood, 5 weeks-True
B. Platelets, 21 days-False
C. FFP- 2 years-True
D. Cryoprecipitate-2 months-False

66.Blood use include


A. Whole blood in trauma-True
B. Packed RBCs in chronic anemia-True
C. FFP in coagulation deficiency-True
D. Platelets in thrombocytopenia-True

67.Changes in stored blood include


A. Raised pH-False
B. Raised PCO2-True
C. Raised K+-True
D. Reduced clotting factors-True

68.Considering osteomyelitis
A. Bone drilling is recommended in acute osteomyelitis-True
B. Sequestrectomy is recommended in Post-acute osteomyelitis-False
C. Amputation has no role in chronic osteomyelitis-False
D. Broad spectrum antibiotic only applicable in acute osteomyelitis-False

69.Management of obstructive uropathy may include


A. Catheterization- True
B. Bilateral orchidectomy-True
C. Endoscopic urethrotomy-True
D. Urethroplasty-True
PREPARED BY MOSES KAZEVU JR

70.In facture healing


A. The periosteal (bridging) callus is composed of different bone from medullary
(gap) callus-False
B. Osteoblast and osteoclast with associated capillary constitute-OSTEON-True
C. Woven bone represents clinical union and lamellar bone radiological union-
True
D. All are true except (b)-False

71.Factors that promote fracture healing


A. Good blood supply for nutrition and oxygen-True
B. Lack of infection-True
C. Excessive movement at fracture site-False
D. All correct expect (c)-True

72.Stored blood
A. Has low pH- True
B. Has high pH-False
C. Has high potassium content-True
D. Has high coagulation factors and low platelets and leucocytes-False

73.Red cell survival in blood storage media


A. Acid, citrate, dextrose (ACD) media is 21 days-True
B. Citrate, phosphate, dextrose (CPD) media is 28 days-True
C. Citrate, phosphate, dextrose, adenosine (CPDA) media is 35 days-True
D. All above are correct-True

74.Patients who present with vomiting of fecal matter is more likely to have
A. Duodenal atresia-False
B. Esophageal ulcer-False
C. Gastric ulcer- False
D. Diverticulum of the ileum- False

75.Patient observation includes


A. Patients gait-True
B. Facial expression-True
C. Paradoxical respiration-True
D. Auscultation of chest-False
PREPARED BY MOSES KAZEVU JR

76.In factures
A. A comminuted fracture is an example of a compound fracture-False
B. A green-stick fracture is commonly seen in both the elderly and the young-
False
C. An oblique fracture occurs due to twisting force-True
D. A pathological fracture occurs secondary to a pre-existing disease of
particular bone-True

77.Collar and cuff sling


A. Is an example of external splint -True
B. Is suitable for displaced humeral fracture-False
C. Can be used to immobilize femoral fractures in children-False
D. Only ‘a’ above correct- True

78.Intramedullary nail is
A. External fixation-False
B. Internal fixation-True
C. Suitable for fracture femoral shaft-True
D. Also known as Kuntscher nail-True

79.Burns are classified by depth as:


A. Superficial partial thickness- True
B. Deep partial thickness-True
C. Full thickness- True
D. Full thickness is also known as second degree burns- False

80.Examination of non-toxic goiter


A. Includes asking the patient to swallow on inspection- True
B. Palpation is best done from the front-False
C. Palpation is best done from behind-True
D. Determining lower edges helps to check for retrosternal goiter-True

81.Patient who presents with goiter causing trachea obstruction


A. Stridor is a presenting feature-True
B. Dyspnea is a presenting feature-True
C. Kocher’s test is usually negative-False
D. Kocher’s test is usually positive-True
PREPARED BY MOSES KAZEVU JR

82.Toxic goiter
A. Exophthalmos is not a feature-False
B. Lid lag is a feature-True
C. Hot and moist skin and palms is a feature-True
D. Rapid pulse rate is not present during sleep-False

83.Patient chest percussion findings include


A. Dullness due to tension pneumothorax-False
B. Dullness due to hemothorax-True
C. Hyper-resonance due to pneumothorax-True
D. Hyper-resonance due to pleural effusion-False

84.Regulation of fluids and electrolytes movement across all membranes is by:


A. Diffusion/filtration-True
B. Active transport-True
C. Osmosis-True
D. All above except (a)-False

85.Atrial natriuretic peptide (ANP) and brain natriuretic peptide (BNP) are
hormones involved in regulation of
A. pH of the body fluids-False
B. Water and electrolyte balance-True
C. ANP is produced in atrium-True
D. BNP is produced in ventricles-True

86.Signs and symptoms of hyperkalemia include:


A. Peaked “T” waves on ECG-True
B. Increased cardiac contractility-False**
C. Muscle weakness-True
D. Respiratory distress-True

87.Treatment of hyperkalemia include


A. Kayexalate-True
B. IV insulin and glucose-True
C. Lasix-True
D. Dialysis if severe-True
PREPARED BY MOSES KAZEVU JR

88.Administration of potassium to correct hypokalemia


A. Give potassium as intravenous bolus-False
B. Not more than 20mEq/h with 40mEq as maximum-True
C. Ensure good urine output-True
D. Cardiac monitor not essential-False

89.Examination of the patient with ascites includes:


A. Shifting dullness-True
B. Succussion splash-False
C. Fluid thrill-True
D. Aspiration of fluid for laboratory examination-True

90.Acute epigastric pain


A. Can be caused by appendicitis-True
B. Can be caused by torsion of the testis-False
C. Can be caused by perforated duodenal ulcer-True
D. Can be caused by ectopic pregnancy in a female patient-False

91.The following cranial nerves originate from midbrain:


A. Optic nerve (CN II)-False
B. Oculomotor nerve (CN III)-True
C. Abducens (CN VI)- False
D. Trochlear nerve (CN IV)-True

92.Secondary thyrotoxicosis
A. Patients are younger than those with primary thyrotoxicosis- False
B. Eye signs (exophthalmos, lid lag) are absent- True
C. Atrial fibrillation is a feature- True
D. Dyspnea is not a clinical feature-False

93.The edge of a malignant ulcer is


A. Shelving-False
B. Rolled up-True
C. Undermined-False
D. Punched out-False
PREPARED BY MOSES KAZEVU JR

94.The following are irritative symptoms of bladder outlet obstruction secondary to


BPH
A. Weak stream-False
B. Nocturia-True
C. Frequency-True
D. Intermittence-False

95.Post-traumatic intracranial space occupying lesion is likely to present with


A. Decreased pulse rate-True
B. Increased blood pressure-True
C. Irregular pulse-False
D. Glasgow coma scale of 2/15-False

96.Headache due to intracranial space occupying lesion


A. Is likely to progressively get worse- True
B. May be associated with early morning vomiting-True
C. May be associated with early morning blurring of vision-True
D. May be associated with decreased blood pressure-False

97.Urinary retention in a 70-year-old man


A. History of increased night frequency is usually present- True
B. History of poor stream of urine is usually present-True
C. There is no history of terminal dribbling-False
D. There is no history of urgency of micturition-False

98.Tachypnea
A. Is respiratory rate of less than 10 per minute in adult patient-False
B. Can be caused by tension pneumothorax-True
C. Can be caused by hemothorax-True
D. Is never associated with tachycardia-False

99.Debridement is
A. The process of making an incision-False
B. The removal of foreign material or dead tissue-True
C. The process of obtaining hemostasis-False
D. None of the above-False
PREPARED BY MOSES KAZEVU JR

100. Factors which affect wound healing include


A. Chronic illness-True
B. Radiation therapy-True
C. Nutrition-True
D. All above correct-True

101. Common causes for rectal bleeding


A. Internal hemorrhoids- True
B. Anal fissure-True
C. Carcinoma rectum- True
D. All correct except “B”-False

102. A direct inguinal hernia would lie in relation to the inferior epigastric artery
A. Medial-True
B. Lateral-False
C. Anterior-False
D. Posterior-False

103. Malignant goiter


A. Medullary carcinoma is the commonest-False
B. Papillary carcinoma is the commonest-True
C. Bone metastasis may occur-True
D. Is slow growing-True**

104. Toxic goiter


A. Patient has good appetite- True
B. Patient gains weight rapidly-False
C. Lid lag is not a feature- False
D. Proximal myopathy is a feature- True

105. Parotid benign tumors


A. Make up 80 to 90% of parotid tumors-True
B. Are pleomorphic adenomas-True
C. Are soft on palpation-False
D. Mostly arise in the superficial lobe-True
PREPARED BY MOSES KAZEVU JR

106. HIV associated sialadenitis of parotid gland


A. Chronic parotitis in children is pathognomic of HIV infection-True
B. Facial disfigurement is a feature-True
C. CT scan shows characteristic swiss cheese appearance of multiple cystic
lesions-True
D. Cosmetic surgery is not advisable-True

107. Signs of advanced cancer of the esophagus include


A. Hoarseness of voice as a sign of involvement of recurrent laryngeal nerve-
True
B. Weight loss of more than 20%-True
C. Absence of palpable supraclavicular lymph nodes-True(from marking scheme)
D. Presence of palpable lymph nodes- True

108. Cancer of esophagus


A. Dysphagia is initially for solids-True
B. Dysphagia is initially for liquids-False
C. Barium swallow X-ray shows a smooth constriction giving a rat-tail
appearance-False
D. Barium swallow X-ray shows irregular and sharp edge constriction giving
shouldering appearance-True

109. Esophagus
A. Below the pulmonary roots the vagus nerves are in contact with esophagus-
True
B. The left trunk of the vagus nerve is on the anterior side of the esophagus -True
C. The right trunk of the vagus nerve is on the posterior side of the esophagus-
True
D. The longest part of esophagus is intra-abdominal-False

110. In acute limb ischemia


A. Thrombo-embolism may be a causative factor- True
B. In-situ blockage of artery by thrombus as factor-True
C. Buerger’s test is positive—True
D. Ankle brachial pressure index useful indicator of severity of ischemia- True
PREPARED BY MOSES KAZEVU JR

111. In lower limb deformities


A. Genu valgus refers to bow legs- False
B. Genu varus refers to knock knees-False
C. Foot drop may follow injury to the common fibular nerve-True
D. All above are correct-False

112. Anterior shoulder dislocation may be reduced by


A. Kocher’s maneuver-True
B. Hippocratic method-True
C. Stimson’s technique-True
D. Only (a) and (b) correct-False
Pioneer in breast surgery-
William Halstead
113. The following factors favor non-union of the fracture
A. Muscle interposition-True Discovered X-rays in 1895-
B. Hematoma at fracture site-True Wilheim Rontgen
C. Poor blood supply-True
Pioneer in abdominal
D. Infection-True
surgery-Theodor Bilroth
114. Protrusion of the tongue is primarily brought about by Pioneer in hip implants- Sir
A. Hypoglossus-False**MS answer John Charnely
B. Genioglossus-True
C. Palatogosssus-False
D. Geniohyoid-False

115. Patients with major burns


A. Are in negative nitrogen balance-True
B. Have normal calorie requirement-False
C. Do not generally become anemic-False
D. Are resistant to septicemia-False

116. Breast cancer is common in women with


A. Positive family history-True
B. History of having breast fed the children-False
C. History of previous radiation exposure-True
D. Having the first child at early age-False
PREPARED BY MOSES KAZEVU JR

117. Ultrasound in a jaundiced patient can demonstrate


A. Choledocholithiasis- True
B. Cholangiocarcinoma-True
C. Intrahepatic biliary dilatation-True
D. Extra hepatic dilatation-True

118. Causes of surgical jaundice include


A. Gall stone-True
B. Massive haemolysis-False
C. Ascaris lumbricoides-True
D. Malignancy of the head of pancreas-True

119. Glasgow coma scale verbal response assessment includes:


A. Incomprehensible sounds- True
B. Confusion-True
C. Withdrawal to pain-False
D. Inappropriate words-True

120. Upper motor neuron lesion


A. Presents with hypotonia-False
B. Presents with hypertonia-True
C. Babinski sign is positive-True
D. Presents with hyporeflexia-False

121. All end colostomies are:


A. Permanent colostomies-True
B. Defunctioning colostomies- True
C. Not suitable for ano-rectal malformation-False
D. Hartmann’s colostomies-True**

122. Cortical bone changes (periostitis) in acute osteomyelitis are evident


A. On admission-False
B. After 2 weeks-True
C. After 2 months-False
D. Only “C” correct-False
PREPARED BY MOSES KAZEVU JR

123. Boil (furuncle)


A. Can occur on any part of the body except the face-False
B. Is infection of hair follicle- True
C. Can cause signs of inflammation-True
D. Surgical drainage is not indicated- False

124. The most frequent anatomic site for squamous cell carcinoma of the
esophagus is
A. Upper third-False
B. Middle third-True
C. Lower third-False
D. Gastro-esophageal junction-False

125. Generalized peritonitis is likely to follow


A. Delayed diagnosis of perforated duodenal ulcer-True
B. Acute pancreatitis-True
C. Rupture appendicitis-True
D. Torsion of the testis- False

126. Parotid gland tumors


A.Most common are benign pleomorphic adenomas- True
B.10% are malignant-True
C.Rapid growth with infiltrating of overlying skin is a sign of high grade
malignancy- True
D. Enlarged palpable cervical lymph nodes are not a feature-False

127. Structures embedded in the parotid gland include


A. Facial nerve- True
B. Retromandibular vein-True
C. External carotid artery-True
D. Parotid lymph nodes-True

128. Perkin’s exercises


A. Are commenced 2 weeks after application of traction kit-False
B. Are commenced within a week after application of traction kit-True
C. Are commenced after 3 weeks when pain at fracture site has subsided-False
D. All above correct except “B”-False
PREPARED BY MOSES KAZEVU JR

129. The main source of bleeding in a scalp laceration is


A. Periosteum-False
B. Loose connective tissue- False
C. Aponeurotica galea-False
D. Dense connective tissue-True

130. Clinical classification of head injury is based on**


A. Mode of injury-False
B. Whether injury is open or closed-True
C. Glasgow coma scale-True
D. Pathological lesion-True

131. Oculomotor nerve lesion presents with


A. Pupil dilatation-True
B. Ophthalmoplegia-False
C. Lateral rectus palsy-False
D. Loss of corneal reflex-False

132. Sensory component of 5th cranial nerve is tested by


A. Wrinkle formation on the forehead-False
B. Touch sensation on the face-True
C. Lifting up shoulder-False
D. Corneal reflex- True

133. When a patient has unilateral vagus nerve lesion and is asked to open the
mouth and say AAAG… The uvula moves and points
A. Away from the lesion-False
B. Towards the lesion-True
C. Stays in the midline-False
D. Elongates within the midline-False
PREPARED BY MOSES KAZEVU JR

134. When a patient has unilateral cranial nerve V motor component lesion
A. Masseter muscles will feel weak on palpation as patient clenches the teeth-
True
B. Temporal muscles will feel well contracted as patient clenches the teeth-False
C. The jaw will deviate to the paralyzed pterygoid muscle when patent opens the
mouth- True
D. The jaw will deviate away from the paralyzed pterygoid muscles when patient
is asked to open the mouth-False

135. Open skull fracture has occurred when there is


A. Cerebrospinal fluid leak from the ear- True
B. Depressed skull fracture under the scalp wound-False
C. Subdural hematoma with superficial bruise on the forehead-False
D. Subarachnoid hemorrhage regardless of the cause-False

136. Patient who presents with acute abdominal pain starting in the umbilical
region and shifts to the right iliac fossa is most likely caused by
A. Perforated duodenal ulcer-False
B. Diverticulitis-False
C. Appendicitis-True
D. Salpingitis-False

137. Murphy’s sign


A. Helps to diagnose ureteric stones-False
B. Helps to diagnose ectopic pregnancy-False
C. Helps to diagnose appendicitis-False
D. Helps to diagnose cholecystitis-True

138. Small intestinal obstruction is likely to present with


A. Radiating pain to the left shoulder-False
B. Spreading pain to the right iliac fossa-False
C. Colic pain in the umbilical region-True
D. Shifting pain to the right iliac fossa-False
PREPARED BY MOSES KAZEVU JR

139. Secondary brain injury


A. Can result from hypoxia-True
B. Can result from hypoperfusion-True
C. Can result from hypoglycemia-True
D. Can result from any of the above-True

140. Shock can be comprehensively defined as


A. A sudden large volume of blood loss-False
B. A diminished effective circulating fluid volume-True
C. A hypotensive state with peripheral vasoconstriction-False
D. An unexpected psychological insult- False

141. Intracranial space occupying lesion (SOL) can be caused by


A. Hemorrhagic traumatic brain injury-True
B. Intracranial sepsis-True
C. Vertebral artery aneurysm- False**
D. Meningioma- True

142. Breast examination


A. Can be done in both sit-up and supine position-True
B. Nipple retraction is not a sign of cancer of the breast-False
C. Peau d’oragne can be sign of cancer of the breast-True
D. Should include palpation of axilla-True

143. Reduced host resistance to infection can be caused by


A. Diabetes mellitus-True
B. Uremia-True
C. Balanced diet-False
D. Acquired immunodeficiency syndrome-True

144. Incisional hernias are related to


A. Wound infections-True
B. Anemia and malnutrition-True
C. Obesity-True
D. The use of absorbable suture material-True
PREPARED BY MOSES KAZEVU JR

145. Empyema thoracic (ET)


A. May be acute or chronic ET-True
B. Pus is thin in acute and thick in chronic ET- True
C. In both acute and chronic ET air entry is reduce-True
D. Air entry is reduced only in acute empyema-False

146. The following are not associated with empyema thoracis


A. Cerebral abscesses- False
B. Amyloidosis-False
C. Nail clubbing-False
D. Kyphoscoliosis-False

147. Chest injuries


A. Tension pneumothorax causes so-called “Paradoxical breathing”- True**
B. May cause a rupture of the diaphragm- True
C. A flail chest results from a wound in the chest wall which acts as a valve that
permits air to enter the pleural cavity but prevents its escape- False
D. May cause pulmonary contusion-True

148. Pathophysiology of shock


A. As perfusion to the tissues is reduced, oxygen deprived cells switch from
aerobic to anaerobic metabolism-True
B. The product of anaerobic respiration is lactic acid leading to systemic
alkalosis-False
C. Decreased perfusion pressure in the kidney leads to reduced filtration at the
glomerulus and an increased urine output- False
D. Vasopressin is released from the hypothalamus and resulting in
vasoconstriction and reabsorption of water in the renal collecting system-True

149. Fibroadenoma
A. Benign tumors with a mixture of stroma and epithelial tissue of the breast-
True
B. Painless, firm, solitary, mobile, slowly growing lump in the breast in a woman
of child-bearing years-True**
C. Leiomyoma in the breast tissue-False
D. Benign tumor of the fibrous tissue in the sweat glands of the breast-False
PREPARED BY MOSES KAZEVU JR

150. The sequence of local examination for fractures is as follows:


A. Look, feel, move-True
B. Inspection, palpation, percussion and auscultation-False
C. Inspection, palpate and then move-True
D. Move, tap, palpate-False

151. The differential diagnosis of an inguinal hernia in a female include


A. Vaginal hydrocele-True
B. Encysted hydrocele of the cord-True**
C. A femoral hernia-True
D. None of the above-False

152. Fournier’s gangrene of the scrotum is associated with


A. Infection- True
B. Minor injuries-True
C. No obvious cause-False*
D. Filariasis-False*

153. Ascites
A. Is an accumulation fluid in the peritoneal cavity- True
B. Exudates from the peritoneal membrane have a high protein content than
transudates and indicate inflammatory or malignant disease-True
C. Transudates from the peritoneal membrane have a higher protein-False
D. Paracentesis is one of the treatment of choice-True*

154. On auscultation of the abdomen


A. Absent bowel sounds suggest paralytic ileus or generalized peritonitis -True
B. High pitched and tinkling bowel sounds suggest sub-acute intestinal
obstruction-True
C. Borborygmi is a rumbling or gurgling sound caused by the movement of gas
and fluids in the intestines-True
D. Succussion splash is performed by having the patient (or a colleague) push
their hands down the midline of the abdomen and then the examiner taps one
flank while feeling on the other flank for the tap-False
PREPARED BY MOSES KAZEVU JR

155. True definition of the signs and symptoms:


A. Xerostomia-Dry mouth- True
B. Dysgusia-altered hearing sensation-False
C. Steatorrhea-pale, fatty, greasy stool-True
D. Anismus-difficulty in passing stool despite prolonged straining-True

156. Concerning tumors, the term “well differentiated” means


A. The cells are so aggressive, wild, and uncontrolled and keep reproducing to
no good end- False
B. Cells and tissue that have mature (specialized) structures and functions-False
C. Cancer cells look more like normal cells under a microscope-True
D. The cells tend to grow and spread more quickly than poorly differentiated
cancer cells-False

157. Diabetic foot has


A. Presence of pathologies such as infection, ulcer and neuropathic
osteoarthropathy-True
B. Is always present in chronic diabetes mellitus-False
C. Any chronic foot ulcer that eventually causes diabetes mellitus- False
D. Patients have an associated reduced ability to feel pain-True

158. A lingual thyroid may present with the following symptoms


A. Dysphagia and pain-True
B. Cervical lymphadenopathy-False
C. Speech impairment and respiratory obstruction-True
D. Severe headache-False

159. Treatment of hyperkalmia include


A. Kayexalate-True
B. Glucose of insulin intravenously-True
C. Lasix-True
D. Dialysis if severe-True
PREPARED BY MOSES KAZEVU JR

160. Post-pelvic fracture urethral stricture is likely to involve


A. Penile urethra-False
B. Bulbous urethra-False
C. Membranous urethra-True
D. Prostatic urethra-False

161. Lymphatic breast drainage


A. Anterior or pectoral group- True
B. Posterior or subscapular group-True
C. Infracalvicular or deltopectoral group-True
D. Central, apical and lateral (brachial) group-True

162. Boundaries of Hasselbach’s triangle


A. Medial half of inguinal ligament-True
B. Linea semilunaris-True
C. Inferior epigastric artery-True
D. All above correct except (b)-False

163. Carotid sheath contains


A. Internal jugular vein-True
B. Carotid artery-True
C. Vagus nerve-True
D. All above correct except (C)-False

164. Extrinsic muscle of the tongue


A. Genioglossus-True
B. Styloglossus- True
C. Geniohyoid-True
D. Hyoglossus-True

165. Contents of femoral triangle (lateral to medial)


A. Nerve, artery, vein-True
B. Artery, nerve, vein-False
C. Vein, nerve, artery-False
D. Only (b) correct-False
PREPARED BY MOSES KAZEVU JR

166. Major causes of thrombosis:


A. Damage to vessel wall-True
B. Alteration in blood flow-True
C. Alteration in constituents of blood-True
D. Above are known as Virchow’s triad-True

167. Types of malignant melanoma


A. Superficial spreading melanoma-True
B. Nodular melanoma-True
C. Lentigo maligna melanoma-True
D. Acral lentiginous and amelanotic melanoma-True

168. Macroscopic features suggestive melanoma


A. Asymmetry-True
B. Border irregularity-True
C. Colour variegation-True
D. Diameter more than 6mm with elevation-True

169. Risk factors of melanoma


A. Xeroderma pigmentosum-True
B. Dysplastic naevus syndrome-True
C. BRAF gene mutation-True
D. Giant congenital pigmented naevus-True

170. Uses of stoma in surgery


A. For feeding-True
B. For lavage-True
C. For exteriorization-False*
D. For decompression, diversion and drainage-True

171. Potential complications of stoma include


A. Bleeding, ischemia, necrosis- True
B. Stenosis, obstruction, retraction-True
C. Herniation, high output fluid loss-True
D. Psychological and psychosexual problems-True
PREPARED BY MOSES KAZEVU JR

172. The following sutures are not absorbable:


A. Steel- True
B. Silk-True
C. Catgut- False
D. Vicryl-False

173. Fibroadenoma of breast


A. Is commonest malignant tumor of the breast- False
B. It has well defined edges with a definite capsule-True
C. Recurrence after excision is the rule-False
D. It is also known as a breast mouse-True

174. Phylloides tumor of the breast


A. Is a subvariety of fibroadenoma- True
B. It has a more stromal than cellular component-False
C. It has a more cellular than stromal component-True
D. May be frankly benign, borderline or undergo malignant transformation-True

175. Cytokines produced by inflammatory cells in fracture healing


A. Recruit more inflammatory cells-True
B. May act as migratory inhibitory factors-True
C. May act as chemotactic factors-True
D. May act as achrating factors-True

176. Regarding compartment syndrome


A. Pain is the least significant symptom-False
B. Active stretch of affected compartment is more reliable than passive stretch
in eliciting pain-False
C. Presence of pulse rules out compartment syndrome-False
D. All above are correct except “C”-False

177. Acute osteomyelitis


A. There is often history of bone trauma-True
B. Long tubular bones are common site-True
C. Bone drilling is preferable to saucerization-True
D. Saucerization is preferable to bone drilling-False
PREPARED BY MOSES KAZEVU JR

178. Chronic osteomyelitis


A. Often is a sequel to acute osteomyelitis-True
B. Bone drilling is as effective as saucerization-False
C. Sequestra and involucrum are a feature-True
D. Discharging sinuses and repeated flare of infection are a feature-True

179. Regarding snake bites-


A. Elepine bite is mainly cytotoxic-False
B. Viper bite is mainly neurotoxic-False
C. Hydrophydae bite is by a sea snake-True
D. All above are correct-False

180. Anti-snake venom


A. Is an antigen-False
B. Is usually polyvalent-True
C. A test dose is mandatory before administering full dose-True
D. All above correct except “B”-False

181. Axillary tail of the breast lies in


A. Upper outer quadrant- True
B. Upper inner quadrant-False
C. Lower outer quadrant-False
D. Lower inner quadrant-False

182. The following cannot affect breast tissue


A. TB-False
B. Syphilis-False
C. Fungal infection-False
D. Parasitic infection-True

183. In Colle’s fracture, distal fragment


A. Anterior displacement-False
B. Ulnar displacement-False
C. Proximal tilt-True
D. Impaction-True
PREPARED BY MOSES KAZEVU JR

184. The following are not complications of chronic osteomyelitis


A. Amyloidosis-False
B. Cerebral abscess-False
C. Pathological fracture-False
D. All above, not associated-False

185. Principles advocating treatment of open tibial fracture include:


A. Antibiotic cover-True
B. Wound debridement-True
C. Fracture stabilization-True
D. Prompt soft tissue cover-True

186. About a fluctuating breast lump it can be


A. Carcinoma-False
B. Lipoma-True
C. Fibroadenoma-False
D. Abscess-True

187. Classical degenerative osteoarthritis note:


A. Osteophytes (lipping)-True
B. Narrow joint space-True
C. Subchondral bone sclerosis-True
D. Subchondral bone cysts-True

188. Torn posterior cruciate ligament diagnosis


A. Lachman test-False
B. Apley’s grinding test-False
C. Posterior drawer test-True
D. All of above correct-False

189. Weber “C” ankle fracture


A. Is fracture below tibio-fibular syndesmosis-False
B. Is fracture at level of tibio-fibular syndesmosis- False
C. Is fracture above tibio-fibular syndesmosis-True
D. Any fracture at ankle joint- False
PREPARED BY MOSES KAZEVU JR

190. Ankle joint mortice


A. Mortice formed by medial and lateral malleolus only-False
B. Mortice formed by medial and lateral malleolus and tibio-fibular
syndesmosis-False
C. Mortice formed by (b) above and articular surface of tibia-True
D. Only (B) above correct-False

191. Regarding blood groups


A. Blood group A+ is associated with increased risk of stomach cancer-True
B. Blood group AB has no plasma antigens but has AB antigens on red cell
surface-True
C. Blood group AB has plasma antigens but no AB antigens on RB cell surface-
False
D. All above are correct except (A)-False

192. Large unilateral left supratentorial space occupying lesion is likely to present
with
A. Dysphasia-True
B. Unequal pupils-True
C. Glasgow coma scale 15/15-False
D. Contralateral hemiplegia-True

193. Brain abscess in the frontal lobe


A. Is most likely a complication of otitis media- False
B. Is most likely a complication of paranasal sinusitis-True
C. Is most likely a complication of infected occipital laceration-False
D. Is most likely a complication of infected cervical laceration-False

194. Head injury patient who presents with skull fracture crossing the midline
A. Is likely to have epidural hematoma from superior sagittal sinus laceration-
True
B. Is likely to have epidural hematoma from middle meningeal artery laceration-
False
C. Is likely to have cerebral spinal fluid leak from both ears-False
D. Is likely to have fracture dislocation of C5/C6 cervical spine-True
PREPARED BY MOSES KAZEVU JR

195. Hemiarthroplasty of hip joint is


A. Replacement of femoral head component only-True
B. Replacement of femoral head component and acetabular cup-False
C. Replacement of only acetabular cup-False
D. Only (B) above correct-False

196. Total hip replacement involves:


A. Replacement of femoral head and acetabular cup-True
B. Replacement of femoral shaft with artificial implant-False
C. Both hips replaced with custom made implants-False
D. Only “C” above correct-False

197. Charnley system is used in


A. Hemiarthroplasty-False
B. Total hip replacement-True
C. In both (a) and (b) above-False
D. None above correct-False

198. Clostridium Welchii is


A. Causative agent of wet gangrene-False
B. Causative agent of gas gangrene-True
C. Aerobe, sporulating, Gram negative bacillus-False
D. Proliferates in dirty wound-True

199. Austin Moore and Thompsons implants are used in


A. Hemiarthroplasty-True
B. Total hip replacement-False
C. In both (a) and (b) above-False
D. None above correct-False

200. Classification of fractures


A. Gastillo and Anderson-Classification of open tibial fractures-True
B. Garden’s classification for fracture neck of femur-True
C. Salter-Harris is a fracture dislocation of the forearm bone-False
D. Galeazzi-Montegia classification of fracture that involves the epiphyseal plate
or growth plate of a bone-False
PREPARED BY MOSES KAZEVU JR

201. Examination for fractures:


A.Use the formula-look, feel, move-True
B.Use the formula-Inspection, palpation, percussion, auscultation-False
C.Always start by looking for signs of shock and associated damage to brain,
spinal cord or viscera-True
D. Always examine motor supply proximal to fracture site- True**

202. In epidural hematoma bleeding can be


A. The middle meningeal artery-True
B. Superior sagittal sinus-True
C. Common carotid artery-False
D. External carotid artery-False

203. Presenting symptoms of urinary stones include


A. Pain as a feature-True
B. Pyuria is not a feature-False
C. Hematuria is not a feature-False
D. Urinary infection is a common feature-True

204. Cancer of the prostate


A. Is a common condition in Zambia-True
B. Residue stone incidence is usually increased-True
C. Anemia is a common feature-True
D. Uremia is not a feature-False

205. Head injury patient with subdural hematoma


A. Is likely to deteriorate faster if blood pressure is Reducing-False**
B. Is likely to deteriorate if blood pressure is Increasing-True
C. Is likely to deteriorate faster if pupils are starting to become Unequal-True
D. Is likely to deteriorate faster if hemiparesis is starting to Present-True

206. Causes of leucopenia


A. Lymphoma-True
B. Severe infections-True
C. Disease of the spleen-True
D. Autoimmune disorders-True
PREPARED BY MOSES KAZEVU JR

207. GoodSall’s rule is related to


A. Anal fissure-False
B. Anal fistula-True
C. Hemorrhoids-False
D. Anal abscesses-False

208. Large bowel intestinal obstruction


A. Early vomiting is a Feature-False
B. Gross abdominal distension is a Feature-True
C. Sigmoid volvulus is a possible Cause-True
D. Rectal carcinoma cannot be the Cause-False

209. Colostomy, defunction colostomy is indicated in the following patients


A. Patient with a third degree or Dukes C cancer of the rectum-False
B. In newborn with Hirschprung’s disease-True
C. In 50 year old male patient with short gun injury to the large bowel-True
D. To protect an anastomosis after sigmoid colectomy-True

210. Upper gastrointestinal hemorrhage can be caused by:


A. Mallory-Weis tear-True
B. Sigmoid volvulus-False
C. Periportal fibrosis due to Bilharzia-True
D. Perforated anterior duodenal ulcer-True

211. Preoperative treatment of toxic goiter


A. Iodine can be given in combination with carbimazole or propranolol-True
B. Iodine can be given and reduces vascularity of goiter-True
C. Nadolol is not a beta-blocker to be used in treating toxic goiter-False
D. It is not necessary to reduce toxic goiter to euthyroid status before operations-
False

212. The following do no contribute to wound disruption


A. Massive blood transfusion-True**
B. Excessive coughing-False
C. Abdominal distension-False
D. Vitamin C deficiency-False
PREPARED BY MOSES KAZEVU JR

213. Cranial nerve that supplies superior oblique which elevates and adducts the
eye is
A. 7th cranial nerve-False
B. 6th cranial nerve-False
C. 5th cranial nerve- False
D. 4th cranial nerve-True

214. Meckel’s diverticulum of the small intestine


A. Is situated at the jejuno-ileal-junction-False
B. Contains all coats of intestinal wall-True
C. May be associated with fibrous band connecting it to the umbilical stump-
True
D. Most commonly presents as diverticulitis-True

215. Regarding mammary glands, the following statements are true:


A. It is a modified lactiferous gland present in the superficial fascia-False
B. Its growth and activity is no under hormonal control-False
C. The suspensory ligament of Cooper are connective tissue septa which define
and interconnect the lobes-True
D. The axillary tail (of Spencer) is its normal extension into the axilla-True

216. Duct papilloma of breast


A. May present as blood stained discharge from the nipple-True
B. It usually arises from a single duct-True
C. Malignant transformation is common in multiple papillomatosis-True
D. Malignant transformation is rare in multiple papillomatosis-False

217. When considering fluid spacing


A. First spacing is normal exchanging between intravascular and extravascular
space-True
B. Second spacing is accumulation fluid into tissues as edema-True
C. Third spacing is accumulation of fluid in body cavities e.g. Ascites, pleural
effusion-True
D. All above correct except “C”-False
PREPARED BY MOSES KAZEVU JR

218. If vagus nerves are severed the respiratory rate **


A. Is increased-False
B. Is decreased-False
C. Remains constant-True
D. Ceases-False

219. In hypernatremia
A. Fever and edema are absent-True
B. Restlessness and raised BP are present-True
C. There is increased urine output-True
D. Only C is correct-False

220. Edges of an ulcer


A. Slopping implies healing ulcer-True
B. Punched out is syphilitic ulcer-True
C. Undermined is Tuberculous ulcer-True
D. Everted edge is carcinoma ulcer-True

221. In the healing of a clean wound the maximum immediate strength of the
wound is reached by
A. 3-4 days-False
B. 5-7 days-False
C. 10-12 days-False
D. 13-24 days-True

222. The following statements about keloids true?


A. They do not extend into normal skin-False
B. Local recurrence is common after excision-True
C. They often undergo malignant change-False
D. They are common in whites than in blacks-False

223. The following factors influence wound healing:


A. Anemia-True
B. Hypoproteinemia-True
C. Vitamin C lack-True
D. Edema-True
PREPARED BY MOSES KAZEVU JR

224. Regarding the splenic artery


A. Largest branch of celiac axis-True
B. Branches to the diaphragm-False
C. Branches to the gastric greater curvature-True
D. Courses along upper border of the pancreas-True

225. The following thyroid tumor has most benign course and good prognosis:
A. Medullary carcinoma-False
B. Papillary carcinoma-True
C. Follicular carcinoma-False**
D. Anaplastic carcinoma-False

226. Cerebrospinal fluid is produced in the


A. Lateral ventricles-True
B. Third ventricle-True
C. Fourth ventricle-True
D. All of the above-True

227. The most common source of cerebral metastases is


A. The breast-False
B. The kidney-False
C. The lung-True
D. Gastrointestinal tract-False

228. Carotid artery occlusion is characterized by the following**


A. Hemiplegia, abrupt or gradual-True
B. Transient motor weakness of one extremity-True
C. Bilateral blindness-False
D. Temporary dysphagia-True

229. The most significant warning of early ischemia after reduction of a


supracondylar fracture is:**
A. Swelling-False
B. Coldness-False
C. Cyanosis or pallor-False
D. Pain distally-True
PREPARED BY MOSES KAZEVU JR

230. Adamantinoma of limb bones are found most frequently in the


A. Humerus-False
B. Femur-False
C. Tibia-True
D. Radius and ulna-False

231. Ewing’s sarcoma


A. Is most common in females-False
B. Prefers the flat bones-False
C. Is multicentric-True
D. Is of low grade malignancy-False

232. Osteogenic sarcoma


A. Commonest malignant bone tumor in the young-True
B. It has preference for knee joint-True
C. X-ray shows Codman triangle and sunray appearance-True
D. All above correct except B-False

233. Classical radiographic features of osteoarthritis


A. Narrowing joint space-True
B. Marginal bone osteophytes-True
C. Subchondral bone sclerosis (eburnation)-True
D. Subchondral bone cysts-True

234. A four-year-old child with a femoral fracture should be immobilized by


A. Skeletal traction-False
B. Russell’s traction-True
C. Bryant’s traction-True
D. Plaster of Paris cast-True

235. Displacement which results in permanent disability in a child if not corrected:


A. Rotation-True
B. Overriding-True
C. Angulation-True
D. Distraction-False**
PREPARED BY MOSES KAZEVU JR

236. The most common location for polyps of the intestine in a child is**
A. Cecum-False
B. Ileum-False
C. Transverse colon-False
D. Rectum-True

237. The ureter is normally constricted


A. At the pelvic-ureteric junction-True
B. Where it crosses iliac vessels-True
C. Where it courses through the bladder wall-True
D. All of the above-True

238. The anatomical order of the renal pedicle from anteriorly posteriorly is
A. Artery, vein, ureter-False
B. Ureter, vein, artery-False
C. Vein, artery, ureter-True
D. Artery, ureter, vein-False

239. The femoral ring which forms base of femoral canal has the following
boundaries:
A. Anteriorly, inguinal ligament- True
B. Posterior, pectineus muscle-False
C. Medially, lacunar ligament-True
D. Laterally, femoral vein-True

240. The 4 portals considered in FAST scan include


A. Pericardiac-True
B. Perihepatic-True
C. Perisplenic-True
D. Pelvic-True

241. Visceral involvement in penetrating abdominal trauma:


A. Liver-40%-True
B. Small bowel-20%-False (its 30%)
C. Diaphragm-40%-False (its 20%)
PREPARED BY MOSES KAZEVU JR

D. Colon-5%-False

242. Visceral involvement in blunt abdominal trauma Visceral involvement in


A. Liver-40%-True penetrating abdominal
B. Spleen-50%-True** trauma Kehr’s sign-
C. Small bowel-10%-False (it is 3%) ruptured spleen
D. Retroperitoneum-15%-False
Murphy’s sign-acute
243. Parietal peritoneum is sensitive to cholecystitis
A. Mechanical stimulus-True Mantrel’s scale- acute
B. Thermal stimulus-True appendicitis
C. Chemical stimulus-True
D. Stretch stimulus-False** Recurrent should
dislocation-
244. Blood storage media stand for: apprehension test
A. ACD: adenine, citrate, dextrose-False
B. CPD: carbon, phosphate, dextrose-False
C. CPDA: carbon, phosphate, dextrose, adenine-False Sliding hernia-hernia en
D. SAGM: saline, adenine, glucose, mannitol-True glissade

245. Open head injury has occurred if Litter’s hernia-Meckel’s


A.There is subarachnoid hemorrhage-False diverticulum
B.There is cerebral spinal fluid leakage from the ear-True Maydl’s hernia- hernia
C.There is depressed skull fracture with scalp laceration with 2 loops bowel
overlying it-True
D. There is neck stiffness on examination-False Ogilvie hernia-
congenital direct hernia
PREPARED BY MOSES KAZEVU JR

246. Diffuse brain axonal injury is **


A. Subcortical injury-True
B. A clinical diagnosis-False Vicryl suture:
C. Associated with decorticate rigidity-True polyglactin
D. Associated with decerebrate rigidity-True Dexon suture-
Polyglycolic acid
247. Regarding malignant melanoma
A. Clarke’s classification deals with level of invasion-True PDS suture-
B. Breslow classification deals with depth of invasion-True polydioxanone
C. Clinical evaluation deals with ABCD evaluation-True Prolene-polypropyelene
D. Only ‘A’ and ‘B; correct-False

248. Melanoma with worst prognosis


A. Amelanotic melanoma-False
B. Nodular melanoma-True
C. Acral lentiginous melanoma-False
D. Superficial spreading melanoma-False

249. The following are causative factors in acute empyema thoracis:


A. Lung infection-True
B. Subdiaphragmatic abscess-True
C. Ruptured esophagus-True
D. Gunshot bullet lodging in pleural cavity-True

250. The following may complicate chronic empyema thoracis


A. Amyloidosis-True
B. Brain abscess-True
C. Broncho-pleural fistula-True
D. Chronic anemia-True
PREPARED BY MOSES KAZEVU JR

251. Failure of healing in empyema thoracis


A. Persistent underlying pathology-True
B. Foreign body in pleural cavity-True
C. Too late starting treatment-True
Viperidae-cytotoxic
D. Inadequate drainage-True
Elapidae-Neurotoxic
252. The following are features of hypocalcemia Colubridae- Hemotoxic
A. Convulsions-True
B. Arrhythmias-True Hydrophydae-Myotoxic
C. Tetany-True
D. Spasms and stridor-True

253. Choice of suture material


A. Is irrelevant-False
B. Depends upon the tissue and area of the body-True
C. Is usually left up to the scrub nurse-False
D. Is whatever the hospital gives you

254. A seroma is also known as


A. A hematoma
B. A chyloma
C. A collection of serum in the tissue
D. All of above correct

255. A white leg is due to


A. Femoral deep vein thrombosis
B. Lymphatic obstruction
C. Femoral vein thrombosis and lymphatic obstruction
D. None of the above

256. Monroe-Kellie doctrine states that


A. Adult skull is a closed box
B. Shifting of intracranial contents occurs with development of space occupying
lesion
C. Blood pressure drops with increasing space occupying lesion-False
D. Pulse rate increases with increase of space occupying lesion-False
PREPARED BY MOSES KAZEVU JR

257. Following head injury, a subcortical lesion involving corona radiata


A. Is likely to develop decerebrate rigidity
B. Is likely to develop cogwheel rigidity
C. Is likely to develop decorticate rigidity
D. Is likely to develop paraplegia

258. First heart sound


A. Is sound of closure of aortic valve- False
B. Is sound of closure of pulmonary valve- False
C. Is sound of closure of mitral and tricuspid valve
D. Is heard at onset of systole- True

259. Post-traumatic mediastinum shift physical examination findings include:


A. Trachea position shift- True
B. Cardiac apex beat position shift-True
C. Chest expansion of more than 5cm
D. Tachypnea-True

260. Investigating a patient who presents with haematemsis include


A. Esophagoscopy- True
B. Gastroduodenoscopy-True
C. Lower midline laparotomy is advisable-False
D. Right subcostal mini-laparotomy is advisable-False

261. Common causes of congenital hydrocephalus include:


A. Brain abscess- False
B. Dandy-Walker cyst- True
C. Arterial venous malformation-True
D. Anold-Chiari malformation-True

262. Massive blood transfusion would mean:


A. >1/2 the patient’s blood volume in 6 hours-True
B. >the whole blood volume in 24 hours-True
C. Is indicated in thrombocytopenia-False
D. Clotting factor deficiency is a complication-True
PREPARED BY MOSES KAZEVU JR

263. Concerning peptic ulcers:


A. All peptic ulcers can be healed using proton pump inhibitors-False
B. Common sites of peptic ulcers are the first part of the duodenum and the lesser
curve of the stomach-True
C. Chronic ulceration will invariably develop malignancy-False
D. Infection with H. pylori is the most important factor in the development of
peptic ulceration- True

264. Pyloric stenosis in adult patient


A. Annulus pancreas can be the cause-False
B. Chronic duodenal ulcer can be the cause-True
C. Merkel’s diverticulum can be the cause-False
D. Cancer of pancreas can be the cause-False**

265. In esophageal achalasia


A. Dysphagia is usually the first symptom-True
B. Pain is rarely a prominent symptom-True
C. There is an absence of peristalsis in the esophagus-True
D. There is absence of ganglion cells in Aurebach’s plexus in the wall of the
esophagus-True

266. Paralytic ileus


A. Is associated with electrolyte imbalance- True
B. May be associated with mechanical intestinal obstruction-False
C. Requires treatment with nasogastric suction and intravenous fluids- True
D. Is associated with retroperitoneal hematoma- False** according to mark scheme i.e. true

267. Rectal carcinoma


A. Symptomatology may include occult blood in stool-True
B. Digital rectal examination is enough to confirm a diagnosis of rectal cancer-
False
C. May present with acute intestinal obstruction-True**
D. In treating the cancer abdominal-perineal resection should always be done-
False
PREPARED BY MOSES KAZEVU JR

268. Neurogenic bladder is likely to be seen in patients with**


A. Quadreplegia-True
B. Paraplegia-True
C. Hemiplegia-True
D. Lower limb monoplegia-False

269. A jaundiced patient who develops pancreatitis is likely to have a gall stone
impacted in:
A. Common hepatic duct-False
B. Csytic duct-False
C. Common bile duct-False
D. Ampulla of vater-True

270. Surgical drainage of an abscess


A. Should be via a small incision with minimal disturbance of the adjacent tissue-
False
B. Should be dependant wherever possible-True
C. Has been out modeled by antibiotic therapy- False
D. Should be undertaken before the signs of fluctuation appear-False

271. Patient with central nervous lesion on observation may have


A. Epigastric peristalsis-False**
B. Ptosis- True
C. Tremors-True
D. Diaphragmatic respiration-True

272. Acute abdominal pain which is


A. Colicky in nature indicates obstruction of a hollow viscus-True
B. Continuous is typically of inflammation-True
C. Maximal in the right loin is typical of duodenal ulceration- False
D. In the right upper quadrant accentuated by inspiration is typically of
cholecystitis- True
PREPARED BY MOSES KAZEVU JR

273. In acute osteomyelitis


A. Saucerization is preferred to bone drilling-False
B. Is a hematogenous infection with staphylococcus aureus as main pathogen-
True
C. Begins in epiphysis-False
D. Can be complicated by metastatic abscesses-True

274. In the management anaphylactic shock, secondary measures include**


A. Adrenaline administration-False
B. Leg end elevation-True
C. Chlorpheniramine administration-False
D. Hydrocortisone intravenous-False

275. Premature termination of treatment with steroids would cause shock in


relation to
A. Cardiac tamponade- False
B. Infection- False
C. Adrenal crisis- True
D. Vagolytic shock-False

276. The following are revised guideline for blood transfusion:**


A. Hemoglobin <10g/dl in patient with cardiovascular diseases-False
B. Hemoglobin <8g/dl-True
C. Blood loss of 10%-False
D. Low potassium levels-False

277. In terms of gastric cancers, precancerous condition includes:


A. Salted food-False**
B. Atrophic gastritis-True
C. Gastric resection-True**
D. Menetrier’s disease-True

278. Surgery is the main modality of the treatment of the cancer of the stomach.
The signs of inoperability include
A. Vomiting-False
B. Krukenberg tumor-True
C. Hard nodular liver-True
PREPARED BY MOSES KAZEVU JR

D. Loss of weight-False

279. Surgical treatment of inguinal hernia


A. Herniotomy-the repair of the weakness of posterior wall- False
B. Hernioplasty- the use of synthetic materials to bridge and strengthen the
posterior wall- True
C. Herniorrhaphy- the opening of the hernia sac-False
D. Herniectomy- the surgical removal of the hernia-False**

280. Signs
A. What is Trousseau’s sign- Migratory thrombophlebitis commonly seen in
carcinoma of pancreas and lung-True
B. Cullen’s sign-discoloration of the flanks- False
C. Grey-Turner’s sign- Cullen’s sign is discoloration due to periumbilical
ecchymosis-False
D. Kleins sign- shifting of pain and tenderness with shifting position-True

281. The indications for intubation in head injury patients include the following
A. Depressed level of consciousness: GCS less than 8-True
B. Need to prevent hyperventilation-False
C. Severe maxillofacial trauma-True
D. Need for pharmacological paralysis for feeding-True

282. The risk factors for developing carcinoma of the thyroid gland include the
following
A. Radiation exposure at a young age-True
B. Hormone replacement therapy-True**
C. Goitre, thyroiditis and other thyroid diseases also predispose to thyroid
cancer-True
D. Oral contraceptives, being obesity, smoking-True**
PREPARED BY MOSES KAZEVU JR

283. Advanced trauma life support (ATLS) is a system that ensures that
A. A patient with trauma is assessed rapidly and accurately-True
B. All the optimum care (primary, secondary and tertiary survey) is given at the
accident site- False
C. In multiple casualties patients are selected for treatment according to best
chance of survival with least expenditure of resources i.e. allow those that will
die and save those that can be saved- True
D. Resuscitation and stabilization of a trauma patient is done according to
priority-True

284. Traumatic space occupying lesion


A. Altered consciousness state is a feature-True
B. Oculomotor palsy is a feature-True
C. Hemiparesis is not a feature-False
D. 6th cranial nerve palsy is not a feature-False

285. Long term sequelae of head injury


A. Brain atrophy is a feature
B. Focal neurologic deficits are not a feature-
C. Memory deficits are not a feature-False
D. Epilepsy is a feature-True

286. Wilms tumor in a child is best treated by:


A. Chemotherapy-True
B. Nephrectomy-True
C. Nephrectomy plus pre-operative irradiation-False
D. Nephrectomy plus post-operative irradiation-True

287. The following are indications for splenectomy


A. Splenic rupture-True
B. Splenorenal shunt-False
C. Idiopathic thrombocytopenic purpura-True
D. Primary splenic tumor-True
PREPARED BY MOSES KAZEVU JR

288. Features of strangulating intestinal obstruction include the following:


A. Little or no abdominal distention-False
B. Abdominal tenderness-True Heller’s operation-
C. Pyrexia over 38oC-True achalasia esophagus
D. Intractable vomiting-True
Halstead operation-
289. Hematoma of the rectus sheath: Radical mastectomy
A. Closely mimics appendicitis-True 5/8 circle- Diameter of
B. Follows strenuous exercise-True needle
C. Anticoagulant therapy predisposes-True
D. Coughing in a debilitated patient-True Tenson band wiring-
Fracture olecranon
290. Treatment of pancreatic pseudocyst include
A. Cyst-gastrostomy-True
B. Cyst-jejunostomy-True Intussusception-
C. Marsupialization-True Sausage shaped mass
D. Anastomosis to gall bladder-False Hypertrophic pyloric
stenosis-olive or marble
291. The roof of the ischiorectal fossa is formed by sized mass
A. Levator ani muscle-True
Linitis plastica-water
B. Deep transverse perineal muscle-False
bottle stomach
C. Obturator internus muscle-False (this forms the lateral wall)
D. External anal sphincter-False (this forms the medial wall with Ectopic gestation-
anal fascia) amenorrhea

292. The minimum calorie requirement for an adult male in bed without fever or
usual loss is
A. 15-20cal/kg body wt-False
B. 25-30 cal/kg body wt-True**
C. 35-50 cal/kg body wt-False
D. 75 cal/kg body wt-False

293. The vagus nerve at the root of the neck is ** (only 1 answer here)
A. Between subclavian artery and vein
B. Behind subclavian artery
C. Anterior to the subclavian vein
D. Behind the cupola (apex) of the lung
PREPARED BY MOSES KAZEVU JR

294. A sliding hernia on the left side **


A. Never contains small bowel-True
B. Always contains small bowel-False
C. Has sigmoid colon as part of the sac-False
D. Only ‘A’ above correct-True Melanoma-hepatomegaly
and glass eye
295. The following are true in pancreatitis Acute appendicitis-
A. There is often glycosuria- obturator test
B. There is an increase in urinary diastase-True
Intussusception- currant
C. There is an increase in serum amylase-True jelly stool
D. Serum calcium may be depressed-True
Transilluminates-hydrocele
296. Patient with raised intracranial pressure
A. Cerebral perfusion pressure is maintained by reduction of arterial press-False
B. Cerebral spinal fluid shifting
C. Uncal herniation is not a feature
D. Coma scale remains unaltered-False

297. The following are components of wound healing except


A. Wound contraction-False
B. Healing by primary intention-True
C. Epithelization-False
D. Scar formation-False

298. Factors affecting wound healing include


A. Jaundice-True
B. Debilitation-True
C. Gender-True
D. Parity-False**

299. Ludwig’s angina precipitating factors


A. Cachexia-True*
B. Chemotherapy-True
C. Dental caries-True
D. Immobility-False
PREPARED BY MOSES KAZEVU JR

300. In a patient with established tetanus, the possible causes of death would
include
A. Aspiration-True
B. Cardiac arrhythmias-True
C. DVT-True
D. Laryngeal spasm-True

301. Tropical ulcers occur in tropical countries. The precipitating factors are:
A. Malnutrition-True
B. Trauma-True
C. Insect bite-True
D. HIV infection-True

302. The following are true of breast cancer


A. Lobular carcinoma is characterized by “Indian File” pattern of tumor cells-
True
B. Lobular carcinoma tends to be familial, multifocal and bilateral-True*
C. Carcinoma in situ implies non-invasive cancer that has not breached basement
membrane-True
D. Macroscopically scirrhous carcinoma is hard, grey and grates on cutting-True

303. About 85-90% radio-opaque urinary stones include the following


A. Calcium oxalate stones-True
B. Pure uric acid stones-False
C. Sodium urate and ammonium stones-False
D. Sulphur containing cysteine-False

304. Complications of acute pancreatitis include the following except:


A. Shock-False
B. Hypocalcemia-False
C. Pleural effusion-False
D. Respiratory failure-False
PREPARED BY MOSES KAZEVU JR

305. Acute pancreatitis can be caused by drugs which include


A. Estrogen-True
B. Diuretics-True
C. Antifungal-True
D. None of the above-False

306. Cervical sympathectomy is procedure which is indicated for the treatment


of**
A. Causalgia-True
B. Raynaud’s disease-True
C. PUD-False
D. Cervical rib-True

307. The following are causes of secondary varicose veins**


A. Klippel Trenauney syndrome-False
B. Oral contraceptive pills-True
C. Pelvic tumors-True
D. Exercise-False

308. Varicose veins are one of the causes of thrombophlebitis other causes include:
A. Blood transfusion-True
B. Enteral feeding-False
C. Chemotherapy-True
D. Trauma-True

309. High risk group in the development of DVT would include the following
features
A. Operation <30 minutes-False
B. Obesity-True
C. >40 years age-True
D. Stroke patient-True

310. The development of skin malignancy, pre-malignant lesions would include:


A. Solar keratosis-True
B. Keloids-False**
C. Radiodermatitis-True
D. Rodent ulcer-True
PREPARED BY MOSES KAZEVU JR

311. An increase in the levels of alkaline phosphatase would be caused by**


A. Biliary cirrhosis-True
B. Metastases in the liver-True
C. Hepatocellular carcinoma-True
D. Hepatitis-True

312. In hepatocellular carcinoma, alpha-fetoprotein levels are increased. What


other conditions would increase the alpha-fetoprotein levels:
A. Carcinoma stomach-False
B. Carcinoma pancreas-False
C. Hepatoma-True
D. Embryonal cell carcinoma of testis-True

313. In gastric outlet obstruction, electrolyte changes:


A. Hypernatremia-False
B. Hyperkalemia-False
C. Hypochloremic alkalosis-True
D. Hypocalcemia-True

314. In etiopathogenesis of carcinoma of the esophagus, precancerous conditions


include
A. Plummer Vinson syndrome-True
B. Achalasia cardia-True
C. Corrosive stricture-True
D. Keratosis palmaris-False

315. Tetany is a condition where there is hyper-excitability of peripheral nerves.


The causes include:
A. Hypoparathyroidism- True
B. Respiratory alkalosis-True
C. Pyloric stenosis-True
D. Chronic renal failure-True
PREPARED BY MOSES KAZEVU JR

316. Medullary carcinoma of the thyroid (MCT)


A. Arises from parafollicular ‘C’ cells-True
B. Associated with MEN I syndrome-False
C. Associated with MEN IIa syndrome-True
D. Sporadic is the commonest-True

317. Causes of pulsatile thyroid swelling: **


A. Follicular carcinoma-True
B. Primary thyrotoxicosis-True
C. Secondary thyrotoxicosis-True
D. Papillary carcinoma-True

318. In hyperthyroidism **
A. Serum creatinine is increased-True
B. Serum cholesterol is increased-False
C. Serum calcitonin is increased-True
D. Serum iodine is reduced-True

319. Frey’s syndrome


A. Auriculotemporal nerve is involved-True
B. Is a complication of submandibular operation-False**
C. Aluminium chloride is part of treatment-False
D. Botulinum toxin injection is treatment-True

320. Arterial blood gas (ABG) analysis is done to assess:


A. Oxygenation status-True
B. Ventilatory status-True
C. Acid-base status-True
D. Electrolyte status-False

321. Etiopathogenesis of malignant melanoma, would include the following


except:**
A. Trauma-False
B. Hypertrophic scar-False
C. Immunosuppression-False
D. Infection-False
PREPARED BY MOSES KAZEVU JR

322. Respiratory alkalosis occurs due to an increase in minute ventilation. Its


features include **
A. Confusion-True
B. Hypokalemia-True
C. Tetany-True
D. Hypotension-False

323. Skin graft is the commonest method of achieving wound cover. The following
are the contraindications for skin graft
A. Avascular wounds-True
B. Small wounds-True
C. Wound near an infected wound with copious discharge-True
D. Beta-hemolytic streptococcus infected wound-True

324. Massive blood transfusion indications


A. >500ml of blood over 5 minutes-False
B. <1/2 of the patient’s blood volume in 6 hours-False
C. >the whole blood volume transfused in 24 hours-True
D. Development of thrombocytopenia-False

325. Complications of pulmonary capillary wedge pressure include


A. Arrhythmias-True
B. Pulmonary hypertension-True
C. Pulmonary infarction-True
D. Pulmonary artery rupture-True

326. Indications of hyperbaric oxygen


A. Arterial insufficiency-True
B. Cancer therapy-True
C. Carbon monoxide poisoning-True
D. Pneumothorax-False

327. Distributive shock can occur in the following situations:


A. Cardiac tamponade-False
B. Septic shock-True
C. Acute adrenal insufficiency-True
D. Tension pneumothorax-False
PREPARED BY MOSES KAZEVU JR

328. Features of class II hemorrhagic shock


A. Loss of 1500-2000ml of blood-False
B. Loss of 800-1500ml of blood-True
C. Loss of 15% of blood-True
D. May be renal shut down-False*
PREPARED BY MOSES KAZEVU JR

SECTION 2: ESSAYS
1. Fractures
A. Fracture classification
Answer:
1. Clinical classification
➢ Open fractures (compound fractures): fracture communicates with the
external environment
➢ Closed (simple fractures): facture does not communicate with the
external environment
2. Anatomical classification: affected bone. It may be proximal, middle or
distal
3. Etiological classification: according to mode of injury
➢ Traumatic fracture: due to trauma in a bone previously free of disease.
It can be direct (fracture present at the site of trauma) or indirect
(fracture at a different site from where trauma was applied)
➢ Stress/fatigue fracture: happens in a normal bone exposed to repeated
heavy loading e.g. athletes, dancers or military personnel
➢ Pathological factures: occurs even with normal stress or trivial trauma
in a diseased bone e.g. in osteoporosis, osteogenesis imperfecta, paget’s
disease or bone cysts/cancer
4. According to radiological pattern:
➢ Transverse
➢ Oblique
➢ Spiral
➢ Comminuted: more than 2 fragments
➢ Compression/crush
➢ Impacted
➢ Greenstick (incomplete break occurring in resilient bones in children)
5. Eponym: according to the person that discovered it or described it
➢ Garden’s classification for fracture of neck of femur
➢ Galeazzi fracture of the radius
➢ Monteggia fractur of the ulna
6. According to displacement
➢ Undisplaced
➢ Displaced
o Translation (shift)
PREPARED BY MOSES KAZEVU JR

o Angulation (tilt)
o Rotation (twist)

B. Describe the phases of fracture healing in a long tubular bone


Answer:
The phases of fractures healing in a long tubular bone include:
➢ Stage of hematoma formation-Immediate
➢ Stage of inflammation -Week 1
➢ Stage of soft callus formation (Fibrocartilaginous callous formation)-
Week 2-3
➢ Stage of bony callous-Weeks 4-12
➢ Stage of remodeling- Months to years

1. Stage of hematoma formation: This begins immediately following the


fracture. Blood vessels supplying the bone and periosteum are ruptured
during the fracture causing a hematoma to form around and within the
fracture. This may be associated with death of some cells, swelling and
pain in the area.
2. Stage of inflammation (Week 1): Within a few hours of the injury there is
an acute inflammatory reaction with influx of various inflammatory cells
from the surrounding soft tissue. There is release of proinflammatory
cytokines e.g. TNF-alpha, IL-1, 6, 11, & 23 which recruit inflammatory
cells to the site of injury, as well as transforming growth factor beta,
platelet derived growth factor. Later on, the clotted hematoma is slowly
absorbed and fine new capillaries begin grow into the area.
3. Stage of soft callus (fibrocartilaginous callous formation) (Week 2-3):
Vascular derived endothelial growth factor (VEGF) causes angiogenesis
at the site within the hematoma and fibrin rich granulation tissue begins to
form. Mesenchymal stem cells are recruited to the area from the
periosteum, the bridged medullary canal and the surrounding muscles.
They differentiate into fibroblasts, chondroblasts and osteoblasts.
Simultaneously osteoclasts start clearing out the dead bone. The
granulation tissue is gradually replaced with fibrous connective tissue and
cartilage. The thick cellular mass with its islands of bone and cartilage
forms the callus or splint on the periosteum and endosteum surfaces. The
fracture ends become “sticky” and movement is reduced. At the same time
woven bone is also laid down.
PREPARED BY MOSES KAZEVU JR

4. Stage of bony callus formation (Week 4-12): cartilaginous callus begins to


undergo endochondral ossification. The cartilaginous callus is resorbed
and begins to calcify. Calcium is laid down in the matrix and the callus
become visible on radiographs. Subperiosteally, woven bone continues to
be laid down, as immature woven bone becomes more densely mineralized
movement and strain at the site decreases progressively. Newly formed
vessels continue to proliferate and allow further migration of mesenchymal
stem cells.
5. Stage of consolidation and bone remodeling (Months to years): Osteoclast
and osteoblasts transform woven bone into lamellar bone. Osteoclasts
remove debris and osteoblasts fill the gap with new bone. The fracture has
consolidated once it has completely healed with bridging bone. Woven
bone is replaced by lamellar bone and the fracture is united. The new bone
is remodeled to resemble normal structure through alternating processes of
bone deposition and bone resorption.

C. List in tabular form complications associated with fractures


Answer:
COMPLICATIONS OF FRACTURES
EARLY LATE
LOCAL COMPLICATIONS
Hemorrhage and hematoma Delayed union
Neurovascular injury Malunion
Visceral injury Non-union
Soft tissue injury Joint stiffness
Hemarthrosis Contractures
Compartment syndrome Avascular necrosis
(Volkmann’s ischemia) Growth arrest
Local Infection Joint deformity
Osteomyelitis
Septic arthritis Osteoarthrosis
Myositis ossificans
SYSTEMIC COMPLICATIONS
Shock Pressure/decubitus ulcers
Fat embolism Hypostatic pneumonia
Thromboembolism (pulmonary
or venous)
PREPARED BY MOSES KAZEVU JR

D. List the methods available for reduction of fractures


Answer:
1. Closed reduction/manipulation
➢ Casting and splintage: sling, black-slab, plaster of Paris
➢ Functional bracing
2. Open reduction with internal fixation
➢ Internal fixation: intramedullary nails (K nails)
➢ External fixation and skeletal traction
3. Traction:
➢ Skin traction
➢ Skeletal traction

2. Questions:
a. Define excised and incised wound
Answer:
➢ Excised wounds are wounds created by the act of cutting out/ severing/
removing undesired tissue from the body.
➢ Incised wounds are wounds created by the act of cutting into tissue in order
to enter or gain access to an internal part of the body. Edges are usually
linear.

b. Describe briefly physiology of healing of excised wound


Answer:
Excised wounds heal by fibrosis and results in a wide scar often hypertrophied
and contracted. Re-epithelialisation occurs from remaining dermal elements
or wound margins.
The stages of wound healing include:
➢ Stage of coagulation/hemostasis
o Happens almost immediately following injury the clotting cascade is
initiated
o Local vasoconstriction occurs and a platelet clot forms
o Platelet derived growth factor is released
➢ Stage of inflammation-Week 1
o Begins after vasodilation
o Inflammatory cells reach site of injury. Initially neutrophils and then
macrophages which secrete fibroblast growth factor for angiogenesis
and skeletal development as well as remove tissue debris.
PREPARED BY MOSES KAZEVU JR

o Lymphocytes secrete inflammatory mediates and oxygen radicals


➢ Stage of proliferation/fibroplasia-Week 2-3
o Lasts 4 to 21 days
o There is formation of granulation tissue
o Fibroblast deposits type III collagen, capillaries provide nutrients.
➢ Stage of remodeling-Week 3-18 months
o Takes 3weeks to 18 months
o There is maturation of collagen by cross-linking which is responsible
for tensile strength of the scar.
o There is also differentiation of fibroblast into Myofibroblast which
contracts the wound.
o Type III collagen is replaced with type I collagen and blood vessels
atrophy.
o Collagenase removes type III collagen and requires zinc as a co-factor

c. List complications of wound healing


Answer:
Early
➢ Wound infection
➢ Septicemia
➢ Septic arthritis
Late
➢ Wound Dehiscence
➢ Contractures
➢ Hypertrophic scars
➢ Keloids
➢ Marjolins ulcer
PREPARED BY MOSES KAZEVU JR

3. Answer the following questions


A. Define a goiter
Answer: a goiter is an enlargement of the thyroid that may be euthyroid,
hypothyroid or hyperthyroid as well as either benign or malignant.

B. Discuss briefly pre-operative investigations for goiter


Answer:
1. Blood investigations:
➢ Serum T3, T4 and TSH: to ensure patient is euthyroid before
thyroidectomy to prevent thyroid storm as a complication.
o Normal serum TSH- 0.4 to 4.0mIU/L, Normal T3=5-12 g/dL,
Normal T4=80-220ng/dl
o An increase in T3 and T4 with a decrease in TSH denotes primary
hyperthyroidism while an increase in TSH, T3 and T4 denotes
secondary hyperthyroidism
o A decrease in T3 and T4 with an increase in TSH denotes primary
hypothyroidism while a decrease in TSH, T3 and T4 denotes
secondary hypothyroidism
o Patient’s TSH, T3 and T4 must be maintained within normal prior
to surgery.
➢ Baseline: urea, electrolytes, creatinine, liver enzymes, full blood count
tests prior to thyroidectomy
o Full blood count: to assess level of hemoglobin ideally should be
>10g/dl, also to check for level of platelets (normal-150 to 400
000/L) for thrombocytopenia platelets can be transfused and
further coagulation studies (INR, PT, aPTT, coagulation time) done
to investigate any underlying bleeding disorder. WBC count (4000-
11000/ L) to rule out any infections before taking patient to
surgery
o Serum urea, electrolytes and creatinine: to rule out any electrolyte
anomalies (sodium, potassium, chloride and calcium) as well as
functioning of the kidneys prior to surgery
o Liver enzymes: AST and ALT should be <35IU/L denoting a
healthy liver
➢ Blood cross match + group and save in anticipated hemorrhage during
thyroidectomy-at least 2 units of blood
2. Imaging
PREPARED BY MOSES KAZEVU JR

➢ Neck X-ray: AP view to check for tracheal deviation, gland


calcification-bleeds easily
➢ Chest X-ray: may show tracheal deviation, lung metastasis and
retrosternal shadow
➢ Ultrasound of the thyroid to check whether it is cystic or solid
➢ ECG and ECHO of the heart- may show arrhythmias with hyperthyroid
goiters
➢ Radioisotope I123 scan: shows either cold, hot or warm nodules
➢ CT scan/MRI: for the anatomy and extent of the goiter as well as
relation to surrounding structures, may show easy of operability.
3. Fine needle aspiration cytology: shows benign or malignant cells
4. Indirect laryngoscopy: to check for vocal cords (intact recurrent laryngeal)
because vocal cord paralysis is a complication of the surgery so prior to
surgery the state of the vocal cords should be inspected so as to know
whether vocal cord paralysis was present prior to the surgery or as a
complication of the surgery.

C. List possible post-operative complications of thyroidectomy


Answer:
Early Intermediate Late
Hemorrhage Infections Hypothyroidism
Hematoma DVT Hypocalcemia with
Injury to recurrent Hypostatic pneumonia pathological fractures
laryngeal nerve Keloids and
Tracheomalacia hypertrophic scars
Upper airway
obstruction
Hypocalcemia
Tracheal perforation
Thyroid storm-Atrial
fibrillation

D. How would you recognize and abort thyroid storm?


Answer:
• During a thyroid storm there will be markedly increased T3 and T4 levels
with symptoms. It is a complication that happens when a patient who is
hyperthyroid has a thyroidectomy performed on them.
PREPARED BY MOSES KAZEVU JR

• Clinical features include: ill, toxic looking, tachycardia, arrhythmias,


pyrexia, tachypnea, sweating profusely, convulsions, confusion, manic
state, coma, diarrhea, vomiting, acute abdomen and heart failure.
Management
• Admit to ICU/high dependency unit a quiet room
• ABCs- secure airway, ensure patient is breathing, administer oxygen and
gain venous access.
• Rehydration: IV fluids and oral fluids
• Control hyperthermia: ice pack/cold compress/sponging/fan, antipyretic-
IV paracetamol)
• Administer beta blocker: Propranolol (use calcium channel blocker e.g.
verapamil or diltiazem in asthmatic or diabetic patients)- this blocks
conversion of T4 to T3 and protects the heart
• Administer antithyroid drugs e.g. propylthiouracil to block T4 and T3
synthesis and release
• Administer glucocorticoids e.g. hydrocortisone or dexamethasone (to
reduce conversion of T4 to T3)
• Administer excess potassium iodide (Lugol’s solution) to decrease T3 and
T4 synthesis
• Administer bile acid sequestrant e.g. cholestyramine to prevent T4 and T3
reabsorption from the gut
• When the patient is stabilized (euthyroid) thyroidectomy can be performed
with appropriate hormone replacement after
• Note: this complication is prevented by making the patient euthyroid prior
to surgery.

E. Discuss briefly clinical presentation of hypoparathyroidism


Answer:
• Parathyroid hormone is the hormone responsible for increasing the levels
of serum calcium. During hypoparathyroidism the levels of calcium in the
blood decrease
• Clinical features that result include:
➢ Neuromuscular excitability
o Carpopedal spasms: tetany (involuntary contraction) of feet or
hands
PREPARED BY MOSES KAZEVU JR

o Chvostek’s sign: facial twitching/contraction of the muscles of the


face are seen on taping the parotid just anterior to tthe tragus
o Trousseau’s sign: carpal tetany in response to a tightened blood
pressure cuff that is pumped above systolic blood pressure
o Paresthesia of peripheries, tongue and mouth
o Cramps and aches
o Airway: laryngospasms and bronchospasm this may cause
respiratory distress and airway obstruction
➢ CNS symptoms
o Seizures
o Papilledema
o Dementia
o Parkinsonism
o Anxiety, depression, fatigue
➢ Other signs (in chronic disease)
o Cataracts
o Dry, coarse, puffy skin
o Reduced tooth growth (if patient is young)

4. Answer the following questions:


A. Classification of head injury
Answer
1. General classification
➢ Open/penetrating
➢ Closed/blunt
2. According to mode of injury (etiological)
➢ Missile injury
o High velocity: gun shot
o Low velocity: catapult, golf ball
➢ Non-missile injury
o High velocity: motor accidents, acceleration/deceleration injury
o Low velocity: during assault, fall or being hit with an object (coup-
contra-coup injury)
3. Pathological classification
➢ Primary head injury: due to trauma can be focal (contusion,
intracerebral hemorrhage) or diffuse (concussion, diffuse axonal injury)
PREPARED BY MOSES KAZEVU JR

➢ Secondary head injury: due to other secondary causes such as hypoxia,


ischemia, hypoglycemia, cerebral edema, cerebral herniation, infection,
hypotension, hypothermia, hyperthermia, hematoma, seizures
4. Clinical classification (using GCS)
➢ GCS 13-15: mild head injury
➢ GCS 9-12: moderate head injury
➢ GCS <9: severe head injury

B. Discuss the value of GCS in head injury


Answer:
• The Glasgow coma scale is a tool that is used clinically to assess the level
of consciousness ranging from 3 to 15. It has 3 parameters namely eye
opening (scored out of 4), best verbal response (scored out of 5) and best
motor response (scored out of 6)
➢ Eye
o Opens eyes spontaneous-4
o Opens in response to verbal stimuli-3
o Opens in response to pain-2
o Does not open-1
➢ Verbal response
o Oriented-5
o Confused conversation-4
o Inappropriate words-3
o Incomprehensible sounds-2
o Not sound-1
➢ Motor response
o Obeys commands-6
o Localizing to pain-5
o Flexion to pain/withdrawing-4
o Decorticate posturing-3
o Decerebrate posturing-2
o No movement-1
• The GCS helps in classifying the severity of head injury (clinical
classification) and need for immediate intervention
➢ GCS 13-15: mild head injury
➢ GCS 9-12: moderate head injury
➢ GCS <9: severe head injury
PREPARED BY MOSES KAZEVU JR

• It thereby
➢ Helps determine need for admission
➢ Helps to determine line of management
➢ Helps in assessing effectiveness of treatment: whether patient is
deteriorating or improving
➢ Helps determine frequency of monitoring patient
➢ Helps estimate the prognosis of the patient (if done early at the onset of
presentation)

5. Discuss briefly the WHO protocol for management of dog bite


Answer:
Treatment depends on the category, the depth of the bite and the nature of the
dog (stray, tamed-vaccinated or tamed-unvaccinated)
Below is a table showing the protocol for post exposure by WHO
CATEGORY BITE STRAY TAMMED TAMED
(VACCINATED) (UNVACCINNATED)
I Lick Reassurance Reassurance Reassurance
II Superficial First aid See vaccination First aid
bite Full ARV certificate Observe patient for 10 days
First aid
Observe patient
and dog for 10
days
III Deep bite First aid First aid First aid
Full ARV ARV (short ARV (full course)
Wound course) Wound debridement
debridement Wound RIG
Rabies debridement
immunoglobulin RIG
(RIG)

Note: ARV= Antirabies vaccine


First aid includes:
• Tetanus toxoid- 0.5ml IM over the deltoid
• Decontamination- wash wound with a lot of water and soap
• Analgesia
• Wound dressing
• Cover on broad spectrum antibiotics: Co-amoxiclav or Doxycycline +
metronidazole for those allergic to penicillins
Antirabies vaccine-provides active immunity
PREPARED BY MOSES KAZEVU JR

• It is a human diploid cell rabies vaccine (HDCV)


• Dosage-1ml over the deltoid or lateral aspect of the thigh as intramuscular
or intradermal (use smaller doses)
• Frequency:
➢ Short course: days 0, 3 and 7
➢ Full course: days 0, 3, 7, 14, and 28
Rabies immunoglobulin-provides passive immunity
• 20IU/Kg body weight
• Half of the dosage is given at the wound site
• Half the remaining dosage is given away from the site of antirabies vaccine
lateral thigh
Prophylaxis is effective when given within 10 days of the bite.

6. Discuss the management of snake bites


Answer:
Treatment in the field/at home
• Crepe bandaging
• Splint limb: this slows down circulation of the toxin (poison)
• Tie a tourniquet (use a venous tourniquet-i.e. not too tight)
• Analgesia
• Avoid cutting, sucking or capping
Treatment in hospital-Team work is of uttermost importance
Initial measures
• Reassure the patient
• Confirm the bite-fangs, site eye witness account
• Ascertain the type of snake, type of bite and first aid given if any
• Resuscitate the patient-ABCs, endotracheal tube may be required with
ventilatory support if needed
• Antishock measures:
➢ Cannulate patient
➢ Input output calculations
➢ Crystalloid fluids-normal saline or ringer’s lactate
➢ Blood transfusion
➢ Steroids
PREPARED BY MOSES KAZEVU JR

• Drugs to be given
➢ Analgesia-pethidine 1mg/kg
➢ Broad spectrum antibiotics
➢ Steroids
➢ Tetanus toxoid 0.5ml IM
➢ Sedation PRN
➢ In cobra bites also give neostigmine 0.5mg IV every 30 minutes
repeated as necessary and atropine 0.6mg to abolish neuromuscular
blockade and paralysis
• Investigations:
➢ Blood
o Full blood count: platelet and Hb
o Coagulation profile: clotting time, bleeding time, PT, aPTT, D-
dimers and fibrin
o Biochemistry: LFTs, U/Es, creatinine, glucose,
creatine phosphokinase (CPK) Indications to give
➢ Immunology-ELISA (snake species) antisnake venom
➢ Urinalysis- RBCs, myoglobinuria, hemoglobinuria, 1. Evidence of
proteins envenomation-
➢ Imaging: Ultrasound and CT scan to assess bleeding, cardiotoxicity
(arrhythmias),
X-ray
coagulopathy (DIC,
• Bite site: bleeding, hematuria),
➢ Irrigation neurotoxicity
➢ Limb elevation (respiratory failure)
➢ Remove the tourniquet 2. Evidence of shock,
hypovolemia
• Anti-snake venom
secondary to diarrhea,
➢ 2 types: Monovalent ASV (species specific) and vomiting and bleeding
Polyvalent ASV (for many species) 3. Evidence of
➢ Give test dose of 0.2ml IM stat (adrenaline, compartment
hydrocortisone and corticosteroids should be on syndrome-
standby for any allergic reactions) progressive limb
swelling
➢ Dose:
o 20ml ASV in 200ml of 5% dextrose or normal saline in 1-2 hours or
o 50ml ASV in 500ml of 5% dextrose or normal saline in 2-4 hours
o Maximum dose depends on clinical and pathological response
o Monitor vitals (temperature, pulse, respiration and blood pressure),
coagulation profile, ABGs and ECG
PREPARED BY MOSES KAZEVU JR

• Surgery
➢ Compartment syndrome-Fasciotomy
➢ Wound debridement + superficial skin graft
➢ Gangrene-amputation

7. An 8-year-old child has 20% burns which occurred at 04:00 hours and arrives in
hospital at 08:00 hours. Calculate the fluid requirement using parkland formula:
Answer:
Estimated weight of child= 2 x (age + 4)
Estimated weight= 2 x (8 + 4) = 24kg

Resuscitation fluid= 24 x 4 x 20= 1600ml (in 24 hours)


Give 800ml of Ringer’s lactate intravenously from 08:00 to 12:00hours to cover
for the first 8 hours when the child was burnt (since 4 hours has already elapsed)
Give 800ml of Ringer’s lactate intravenously in remaining 16 hours (from
12:00hours to 04:00 next day)

Maintenance fluid
100ml/kg for first 10kg
50ml/kg for next 10kg
20ml/kg for remaining 5kg

Maintenance fluid= (100 x 10) + (50 x 10) + (20 x4)= 1580ml


Give 1580ml in 24 hours dextrose-saline (5% dextrose)
PREPARED BY MOSES KAZEVU JR

8. An 8-year-old child weighing 25kg has 20% burns which occurred at 04:00 hours
and arrives in hospital at 08:00hrs. Calculate fluid requirement using Parkland
formula:
A. Resuscitation fluid
Answer:
Volume of fluid in 24 hours= 4ml x Mass (kg) x % burns
Volume of fluid in 24 hours= 4 x 25 x 20
Volume of fluid in 24 hours= 2000ml
Give:
➢ 1000ml of Ringer’s Lactate Intravenously in 4hours (from 08:00 to
12:00hours) since child was burnt at 04:00 hours so 4hours from 8hours
of first fluid has elapsed.
➢ 1000ml of Ringer’s Lactate intravenously in remaining 16hours (from
12:00hours to 04:00 next day).
B. Maintenance fluid
Answer:
Maintenance fluid:
• 100ml/kg for first 10kg
• 50ml/kg for next 10kg
• 20ml/kg for the remaining 5kg
• Total fluids in 24 hours= (100 x 10) + (50 x 10) + (20 x 5)
• Total fluids in 24 hours= 1600ml
• Give 1600ml in 24 hours dextrose-saline (5% dextrose)

9. How much fluid are you going to give a 3-year old child weighing 15Kg with
20% burns in the first 48 hours?
Answer:

Volume of fluid in the first 24 hours= 4ml x Weight x % burns


Volume of fluid in the first 24 hours= 4 x 20 x 15
Volume of fluid in the first 24 hours= 1200ml

In first 24 hours give 600ml of Ringer’s lactate to run intravenously over 8 hours
and 600ml of Ringer’s lactate to run in the remaining 16 hours

Maintenance= (100x10) + (50 x 5)


Maintenance= 1250ml of dextrose-saline in 24 hours on day 2.
PREPARED BY MOSES KAZEVU JR

10.Discuss the immediate management of a 5-year-old child with 15% burns.


Answer:
Admit patient to burns ward
ABCs
➢ Secure airway- suction any secretions, assess for need for intubation or
tracheostomy especially for inhalational burns
➢ Ensure child is breathing and give child oxygen
➢ Gain venous access (cannulate child)- send blood for group and save, Full
blood count, Urea, Electrolytes and Creatinine, and Liver enzymes.
Catheterize patient, monitor urine output (input/output chart) and Start fluids
➢ Estimated weight of child= 2 x (age + 4) = 2 x (5 + 4) =18kg
➢ Resuscitation fluid
o Volume in first 24 hours= 4ml x 15 x (estimated weight of child
approximately=18kg)
o Volume in first 24 hours= 1080ml
o Give 540 ml in first 8 hours from time of burn injury and 540ml in next
16hours
o Fluid of choice= Ringer’s lactate
➢ Maintenance fluid
o Volume= (100 x 10) + (50 x 8)
o Volume in next 24 hours= 1400ml
o Fluid of choice Dextrose 5%
Drugs given
➢ Analgesia
➢ Tetanus toxoid 0.5ml
➢ Broad spectrum antibiotics
Monitor vitals hourly
Daily wound cleaning with application of silver sulphadiazine
PREPARED BY MOSES KAZEVU JR

11.Write short notes on assessment of a patient with burns


Answer:
➢ History:
o Time and mechanism of the injury
 Time the person was burnt
 Type of burn i.e. scald (what was the liquid, if tea/coffee was there
milk in it), flame, electrical (what voltage, was there a flash/arching,
what was the contact time) or chemical (Acid or alkaline, how much
exposure?)
o Circumstances around the burn
 How was the person put out?
 How long were they alight for?
 What first aid was carried out? If cooling was performed, what with
and for how long?
 Concomitant injuries (i.e. fall from height, road traffic accident,
explosions)
 Likelihood of inhalational injury (did the event occur in an enclosed
space?)
 Any comorbidities present
 Any suspicion of non-accidental injury?
o Allergies, medication, past/present medical history, last meal
➢ Physical assessment:
o Assess burn percentage and severity using:
 Lund and Browder chart (most accurate)
 Wallace Rule of nines (in adults): Head and neck (9%), Upper limbs
(9% each), Lower limbs (18% each) Trunk-36% (18% in front and
18% at the back) and Perineum-1%
 Rule of sevens (in children): Head and neck (28%), Upper limbs (7%
each), Lower limbs (14% each) Trunk-28% (14% in front and 14%
at the back) and Perineum-2%
 Rule of palms (the palms represents 1% of total body surface area
o Examine systems:
 Neurological examination: look for alteration of mental status
(confusion)
PREPARED BY MOSES KAZEVU JR

 Respiratory examination: look out for signs of inhalational burns


(soot in mouth or nostrils, burnt nasal hair, stridor, respiratory
distress, circumferential burns on chest)
 Cardiovascular system: look for signs of shock (tachycardia,
hypotension)
 GIT examination
 Musculoskeletal and skin: look for fractures and assess skin for burn
percentage area
➢ Investigations:
o Full blood count to pick up any cytopenias
o Group and save/ Cross match
o Urea, electrolytes and creatinine to assess renal function and electrolyte
imbalance
o Liver enzymes and function tests
o Pus swab
o For electrical burns: 12 lead ECG monitoring and cardiac enzymes
o For inhalation burns: chest X-ray and Arterial blood gases

12.Discuss briefly the investigations in obstructive jaundice


Answer:
Blood investigations
➢ Liver function tests
o Serum bilirubin: Both direct and total bilirubin should be assessed.
Normal bilirubin levels are <1mg/dl i.e. 0.8-1.2mg/dl (17mol/L). In
obstructive jaundice serum bilirubin >10mg/dl and direct bilirubin
>20% of total bilirubin in obstructive jaundice.
o Prothrombin time: normal is 12-16 seconds. It is significant if it is
more than 4 times the control.
➢ Liver and hepatobiliary enzymes
o 5’ nucleotidase-this is the most reliable since its level is not influenced
by bone disease and enzyme is not induced by alcohol. It will be
elevated.
o Gamma glutamyltransferase (GGT): is elevated
o ALP (normal 3-12KA) it >30KA in obstructive jaundice
o ALT and AST are minimal or sometimes not elevated (<35IU/L)
➢ Full blood count: neutrophilia may be seen in some inflammatory
conditions
PREPARED BY MOSES KAZEVU JR

➢ Tumor markers
o CA 19/9: >70units/L in CA pancreas
o Alpha fetoprotein: elevated in hepatocellular carcinoma
Imaging:
➢ Abdominal ultrasound: may show gallstones
➢ Endoscopic retrograde cholangiopancreatography (ERCP): To visualize
site of obstruction, brush biopsy or bile sample analysis. It is both
diagnostic and therapeutic
➢ Visualize obstruction and anatomical relations:
o Magnetic resonance cholangiopancreatography (MRCP)
o CT scan
➢ Percutaneous transhepatic cholangiography (PTC): decompress, assess
proximal dilated obstructed biliary system if ERCP fails.
➢ Endoscopic ultrasound: to assess pancreatic mass, staging disease and
identify involvement of portal venous system, common bile duct stones
Urine
➢ Urinalysis: urine appears dark and may show presence of conjugated
bilirubin

13.A 60-year old man present with bloody urine for 2 weeks.
A. What are the causes of hematuria?
Answer:
1. Kidney causes:
➢ Pyelonephritis
➢ Glomerulonephritis
➢ Renal stones
➢ Tuberculosis of the kidney
➢ Trauma
➢ Polycystic kidney disease
➢ Renal tumor
2. Ureteric causes
➢ Ureteric stones
➢ Ureteric tumor
3. Bladder causes
➢ Cystitis
➢ Tuberculosis of the bladder
➢ Bladder calculi
PREPARED BY MOSES KAZEVU JR

➢ Benign prostatic hyperplasia


➢ Prostate carcinoma
➢ Prostatitis
➢ Bladder cancer
4. Urethral
➢ Urethritis
➢ Urethral injury
➢ Urethral tumors
5. Others: bleeding disorders (thrombocytopenia), Leukemia

B. Give an outline of investigations for hematuria


Answer:
1. Urine
➢ Urinalysis: confirm hematuria, urinary tract infection, proteinuria
➢ Urine microscope, culture and sensitivity: urinary tract infection,
WBCs, RBC casts, hyaline casts
➢ Urinary cytology: urothelial malignancy
➢ Urine LAM: Tuberculosis
2. Blood
➢ Full blood count: thrombocytopenia and urinary tract infections
(increased WBC)
➢ Coagulation studies for blood dyscrasias: Bleeding time, clotting time,
prothrombin time, activated partial thromboplastin time
➢ Urea and creatinine: kidney function
➢ Prostatic specific antigen- Benign prostatic hypertrophy and prostate
cancer. >10ng/ml rise is significant
3. Imaging
➢ Cystourethroscopy: to look for bladder or urethral pathology
➢ Kidney, ureter and bladder Xray: urolithiasis
➢ Ultrasound: polycystic kidney disease
➢ Intravenous urogram: function of the kidneys
PREPARED BY MOSES KAZEVU JR

14.Answer the following question


A. Define osteoarthritis
Answer: This is a degenerative bone and joint conditions initially involving
thinning of the hyaline cartilage until the underlying bone is exposed and
becomes damaged due to direct stress

B. Discuss briefly etiological factors in general and clinical presentation of


osteoarthritis of lumbar spine
Answer
Etiological factors:
1. Advancing age
2. Gender: it is more common in post-menopausal women
3. Spinal trauma/mechanical strain: obesity, trauma, occupation involving
repetitive stress and damage to the joints and bone
4. Underlying joint or cartilage disorders: inflammatory arthritides, Paget
disease and avascular necrosis
5. Metabolic disorder: Hemochromatosis and Wilson disease
6. Hemoglobinopathies e.g. sickle cell disease, thalassemia
7. Genetics (family history)

Clinical presentation
• Lower back pain and joint stiffness: initially pain and stiffness occur after
periods of inactivity however later in disease process pain and stiffness is
worse with physical activity and prolonged sitting
• Limited range of moving during flexion and rotation of spine
• Swelling and tenderness over the affected vertebrae
• Radicular pain from spinal stenosis e.g. with sciatica: there may be pain
and numbness in the legs if the nerves are compressed
• Acquired spondylolisthesis

C. List the four classical radiological findings in osteoarthritis


Answer:
1. Loss of joint space (asymmetrical)
2. Osteophytes/ bone lipping
3. Subchondral bone sclerosis
4. Subchondral bone cysts
PREPARED BY MOSES KAZEVU JR

D. List the treatment modalities available for osteoarthritis


Answer:
• Medical treatment
➢ Non-pharmacological: exercise, weight loss, walking aids, heat/cold
therapy and physiotherapy
➢ Pharmacological:
o Analgesics:
▪ Oral NSAIDs/paracetamol
▪ Topical NSAID gel/spray e.g. Dynapar
▪ Topical capsaicin
o Intra-articular steroid injections
• Surgical treatment
➢ Arthroscopy
➢ Osteotomy
➢ Athroplasty
➢ Fusion

15.Concerning thrombosis answer the following questions


A. Virchow’s triad
Answer:
1. Endothelial damage (venous damage)
2. Stasis of blood flow (venous stasis)
3. Hypercoagulability

B. Risk factors for thrombosis


Answer:
Non-modifiable risk factors
• Age>60 years
• Past history or family history of venous thromboembolism

Modifiable risk factors


• Immobility (venous stasis)
➢ Recent surgery
➢ Recent travel
➢ Occupation: truck drivers, officer workers
• Malignancies e.g. prostate and ovarian (activation of thrombin)
PREPARED BY MOSES KAZEVU JR

• High levels of estrogen: combined oral contraceptive pills, pregnancy,


hormone replacement therapy, obesity (BMI >30)
• Blood dyscracias: thrombophilia
• Significant medical comorbidities: vasculitis, heart disease-atrial
fibrillation, nephrotic syndrome, polycythemia rubra vera

C. Major complications of DVT


Answer:
• Thrombophlebitis/cellulitis
• Compartment syndrome
• Gangrene
• Post-thrombotic syndrome (long term pain, rash, swelling and redness)
• Pulmonary embolism
• Cerebrovascular accident/Stroke (paradoxical emboli with heart defect
such as patent foramen ovale or Atrial septal defect)
• Chronic venous insufficiency

16.Principles of management for gas gangrene


Answer:
• Medical therapy
➢ Oxygenation, Intravenous fluids and correction electrolyte
abnormalities
➢ Administer tetanus toxoid if indicated
➢ Administer intravenous broad-spectrum antibiotics (e.g. penicillin+
clindamycin, vancomycin or linezolid plus piperacillin-tazobactam or
a cabapenem or ceftriaxone and metronidazole) and analgesia
➢ Hyperbaric oxygen therapy
➢ Use of hydrogen peroxide for wound care
• Surgical therapy
➢ Wound debridement and excision of dead tissue
➢ Amputation
PREPARED BY MOSES KAZEVU JR

17.Lower urinary tract symptoms


Answer
• Irritative symptoms
➢ Increased frequency
➢ Increased urgency
➢ Urge incontinence
➢ Nocturia
➢ Dysuria
➢ Hematuria
• Obstructive symptoms
➢ Poor urinary stream
➢ Hesitancy
➢ Intermittency
➢ Terminal dribbling
➢ Urinary retention
➢ Straining to void
➢ Incomplete voiding
➢ Overflow incontinence

18.Answer the following questions


A. Examination of an ulcer
Answer:
• Greet patient, introduce self to patient, explain the examination about to be
undertaken and gain consent
• Place patient in bed and in anatomical position
• General examination:
➢ General condition: stable or deteriorating
➢ Nutritional status: good, fair or poor
➢ Vitals: temperature, pulse, blood pressure, oxygen saturation and
respiratory rate
➢ Jaundice, pallor, cyanosis, finger clubbing and lymph adenopathy
• Local examination
➢ Inspection: (from foot end of the bed then right side)
o Site of ulcer: digits (Arterial), venous ulcer (medial malleolus),
arterial ulcer (lateral malleolus), trophic ulcer (heel/pressure point)
o Number of ulcers: single or multiple
o Size of ulcer: dimensions of ulcer
PREPARED BY MOSES KAZEVU JR

oShape of ulcer: regular/irregular/well defined/ill-defined.


oDepth of ulcer: superficial or deep
oMargin: regular/irregular/well defined/ill-defined
oEdge: slopping (healing ulcer), undermined (tuberculous ulcer),
punched out (syphilitic ulcer), elevated/everted/rolled out
(malignant-basal cell or squamous cell ulcer), raised and beaded
(rodent-basal cell carcinoma)
o Floor: red (healing), slough (non-healing), wash leather slough
(syphilitic), pigmented (melanoma or pigmented basal cell
carcinoma)
o Discharge: serous (healing), purulent (infected), bloody
(malignant), seropurulent, serosanguinous
o Surrounding area: inflammation, edema, scarring, pigmentation,
excoriation
➢ Palpation:
o Tenderness of edge, base and surrounding area
o Warmness over surrounding area
o Edge for induration
o Base for induration and mobility (fixed-malignant, mobile-non-
malignant)
o Depth of ulcer: palpation for deeper structures and ulcer relation to
tissue
o Bleeding on palpation and touch
o Surrounding skin
o Adjacent joint mobility
o Peripheral arterial pulse peripherally in relation to ulcer
o Regional lymph nodes: tender (acute infection), hard and fixed
(carcinoma), sinus (nonspecific TB or carcinoma)
➢ Auscultation: of peripheral arteries for any bruits.
• Thank the patient
PREPARED BY MOSES KAZEVU JR

B. Examination of abdominal pain


Answer:
• Greet patient, introduce self to patient, explain the examination about to be
undertaken and gain consent
• Place patient in bed and in anatomical position
• General examination:
➢ General condition: stable or deteriorating. Is patient restless
(Abdominal colic) or lying still in bed (peritonitis)
➢ Nutritional status: good, fair or poor
➢ Vitals: temperature, pulse, blood pressure, oxygen saturation and
respiratory rate
➢ Jaundice, pallor (conjunctival and palmar), cyanosis, finger clubbing,
lymphadenopathy and signs of dehydration
• Abdominal examination: adequate exposure (from nipple to half way of
the thigh)
➢ Inspection (from foot end and then right side of the bed)
o Scars, markings or masses
o Abdominal contours
o Abdominal distension
o Flank fullness
o Symmetry
o Movement of abdomen with respiration
o Cough impulse: ask patient to cough and observe their facial
expression
➢ Palpation: enquire where the patient is experiencing the pain and start
palpation in the quadrant furthest from the pain.
o Light palpation for tenderness, guarding (peritonism) and masses
o Deep palpation: for tenderness, masses and organomegaly
▪ Palpate the liver and estimate its length below the costal margin
▪ Palpate spleen
▪ Abdominal aorta
▪ Ballot kidneys
▪ For masses assess the size, shape and consistency
o Rebound tenderness
➢ Percussion
o Abdomen
o Check for shifting dullness and fluid thrill
PREPARED BY MOSES KAZEVU JR

➢ Auscultation of bowel sounds (some prefer doing this before palpation


to avoid stimulation of the bowel) and renal artery bruits
➢ Examination of the groin, genitals and hernia orifices
➢ Digital rectal examination: anal tone, blood, masses, prostate
enlargement (males)
• Thank the patient and cover them up

C. Examination of a breast lump


Answer:
• Greet patient, introduce self to patient, explain the examination about to be
undertaken and gain consent
• General examination:
➢ General condition: stable or deteriorating
➢ Nutritional status: good, fair or poor
➢ Vitals: temperature, pulse, blood pressure, oxygen saturation and
respiratory rate
➢ Jaundice, pallor, cyanosis, finger clubbing and lymph adenopathy
• Local examination
➢ Before beginning the local examination ask which breast has the
lump(s) and if there is any discharge
➢ Adequate exposure full exposure of the chest and the patient initially is
asked to sit upright on the side of the bed
➢ Inspection: breast contours, breast symmetry/asymmetry, scars,
marking, obvious masses, swelling, skin changes (erythema, Peau de
’orange, ulceration), dimpling, nipple contours, nipple retraction,
nipple ulceration or nipple discharge. Inspection should be done with
the following maneuvers
o Sat in an upright position with arms on the side laying on the thighs
o Sat with arms pressed against the hips and pushed inwards
o Sat with arms over the head while leaning forward
➢ Palpation: ask the patient which breast has the lump(s) and examine
that breast last. Ask patient to lay flat on the bed with the arm on the
side of which the breast is to be examined placed under the patient’s
head.
o Palpate the 4 quadrants and axillary tail in clockwise or spiral
manner making concentric circles with one hand on top of the other
will varying pressure for light, medium and deep palpation.
PREPARED BY MOSES KAZEVU JR

o Note consistency, tenderness and for the lump (position, size,


shape, consistence, fluctuation, mobility and overlying skin)
o Also palpate the nipple and express any nipple discharge (the
patient may be asked to squeeze the nipple to demonstrate
discharge if any has been noticed)
o Palpate the axillary lymph nodes
• Thank the patient and cover them up

19.Write short notes on


A. Autologous blood transfusion, types, uses and limitations
Answer:
• Autologous blood transfusion is a process wherein a person receives their
own blood for a transfusion instead of banked allogenic (separate donor
blood)
• There types of autologous blood transfusion include:
➢ Preoperative Autologously donation: blood is collected in advance of
an elective procedure, stored in the blood and transfused back to the
patient when required.
➢ Blood/cell salvage: blood collected from suction, surgical drains or
both is filtered/washed and re-transfused back into the patient. This can
be done intraoperatively, postoperatively or both
➢ Acute normovolemic hemodiluation: blood is collected immediately
prior to surgery and blood volume restored by crystalloid or colloid.
The blood is then re-transfused towards the end of surgery once
hemostasis is achieved
• Uses:
➢ In patients with rare blood groups
➢ Patients with multiple antibodies
➢ In Jehovah’s Witnesses
➢ Patients with serious anxieties about transfusion of allogeneic blood
➢ In patients with high initial hemoglobin level
➢ Substantial blood loss anticipated with surgery e.g. during major
procedures- cardiothoracic, vascular, orthopedic neuro and
transplantation surgeries
• Limitations
➢ Preoperative autologous donation
PREPARED BY MOSES KAZEVU JR

o Not all patients are able to tolerate the preoperative donation


process (not recommended for patients with anemia, cyanotic heart
disease, ischemic heart disease, aortic stenosis or uncontrolled
hypertension)
o Limitation to the amount of blood that can be predeposited
o Blood is wasted if the patient does not need a perioperative
transfusion as it cannot be administered to other patients.
o Patients sometimes require more blood in excess to the
predeposited units
➢ Blood/Cell salvage
o Requires complex specialized equipment and this is expensive
o Depletion of plasma and platelets as washed autotransfusion system
removes the plasma and platelets to eliminated activated clotting
factors and activated platelets.
o Cannot be used in the presence of bacterial contamination or
malignancy
o Blood salvaged may contain cell debris, free hemoglobin and
micro-aggregates
o Electrolyte imbalance (hypernatremia, hyperchloremia,
hypomagnesemia, hypocalcemia), hypoalbuminemia and DIC can
occur with large volumes of salvaged blood.
o Incorrect use of washing and filtration devices may result in red cell
destruction and air embolism.
➢ Acute normovolemic dilution
o Acute and significant reduction in hematocrit may lead to
hemodynamic instability and myocardial ischemia in susceptible
patients

B. Preoperative transfusion check list


Answer:
1. Ensure two appropriate medical personnel are present to verify blood
2. Check blood unit: integrity of blood pack, ABO and blood group of the
donor blood, batch number, date of collection, expiry date, what the blood
has been screened for e.g. HIV, Hepatitis B and C, syphilis
3. Patient: demographics (name, gender, age), file number, indication for
transfusion, amount (units) to be transfused, patient’s ABO and Rh blood
PREPARED BY MOSES KAZEVU JR

group (check whether blood has been collected from patient and that
patient has been crossmatched), informed consent gained
4. Verify that donor blood is compatible with patient
5. Ensure all necessary equipment and personnel for transfusion are present
e.g. giving set with 2 filters and personnel to monitor vital.
6. Transfuse slowly over 4 hours
7. Give furosemide
8. Monitor vitals
9. Check if there is any transfusion reaction, if present stop transfusion, infuse
normal saline (use a new IV line), check and monitor vital, check that the
right pack has been given to the right patient, notify medical officer, draw
a fresh sample to send back to the lab for re-typing and cross matching

C. Steps to be taken in aborting hemolytic blood transfusion reaction


Answer:
• Stop transfusion immediately and inform medical officer on call.
• Check and monitor vital signs: give oxygen if oxygen stats <92% (patient
may also need intubation)
• Maintain intravenous access (do not flush existing line and use a new IV
line if required)
• Infuse normal saline
• Examine the blood to determine if the patient received the correct blood
unit
• Obtain fresh samples of patient’s blood (clotted and in EDTA) for repeat
cross-matching and serological testing. This should be sent back to the lab
along with the freshly collected blood sample.
• Send blood also for electrolytes, urea, free hemoglobin and coagulation
screen.
• Catheterize patient and administer furosemide IV large dose (150mg) if
this fails use 20% mannitol (100ml)
• Low dose dopamine may be considered to increase renal blood flow
• Treat DIC if present with fresh frozen plasma and platelets
• Treat hyperkalemia if present: 50ml of 50% dextrose + 10IU insulin for
potassium >6.0mmol/L
PREPARED BY MOSES KAZEVU JR

20.Write short notes on:


a. Salter-Harris classification of epiphyseal
fractures
Answer:
Salter and Harris (1963) described 5 different
types of epiphyseal fractures.
➢ Type 1 (“Straight through”)-5%
o Fracture involves a transverse fracture
through the hypertrophic or calcified zone
of the epiphyseal plate
➢ Type 2 (“Above”)-75% (commonest)
o Fracture involves growth plate; it extends
into metaphysis above/ away from the
joint but spares the epiphysis
o Fracture often splits off a triangular
metaphyseal fragment of bone (sometimes
referred to as a Thurston-Holland
fragment)
➢ Type 3 (“Lower”)-10%
o The fracture involves the growth plate and
epiphysis below the joint but spares the
metaphysis
➢ Type 4 (“Through everything”)-10%
o Fracture involves the growth plate,
metaphysis and epiphysis (Fracture forms a
T)
➢ Type 5 (“Rammed/Crushed”)-1%
o Due to longitudinal compression injury of
the epiphyseal plate.
o There is no visible fracture but growth plate
is crushed and this may arrest growth
PREPARED BY MOSES KAZEVU JR

b. Gatland classification of supracondylar fractures


Answer:
The Gatland classification is divided into 3 types (based on displacement)
➢ Type 1:
o This is an undisplaced supracondylar fracture
o Treatment: collar-cuff or backslab
➢ Type 2:
o Angulated fracture with posterior cortex still intact (periosteal hinge)
o Subdivided into:
▪ Type IIA- less severe injury with distal fragment angulated
▪ Type IIB- severe injury, distal fragments angulated and
malrotated
o Treatment: manipulation under anesthesia and collar cuff or backslab
(convert Type II to type I)
➢ Type 3:
o Completely displaced fracture (although the posterior periosteum is
usually still preserved, which is usually still preserved, which will
assist surgical reduction)
o Treatment: manipulation under anesthesia, K-wire (Kirschna wire)
fixation, after 3 weeks take the patient to theater to remove the K-wires
under general anesthesia and put a slab

c. Draw and label patterns of fracture of neck of femur


Answer:
Fractures can be viewed based on
➢ Site of fracture (Anatomical classification)
➢ Displacement (Garden’s classification)
Site of fracture, include: Subcapital, Transcervical and Basal
PREPARED BY MOSES KAZEVU JR

Garden’s classification:
• Type 1: Incomplete fracture + minimal displacement
• Type 2: complete fracture + no displacement
• Type 3: complete fracture + partial displacement
• Type 4: complete fracture + complete displacement
PREPARED BY MOSES KAZEVU JR

21.Classification of abscesses
Answer:
• According to depth/ location
➢ Superficial/skin abscess: cutaneous or subcutaneous e.g. carbuncle
➢ Deep/internal abscess e.g. lung abscess
• According to features of presence/absence of typical inflammatory signs
➢ Hot abscess (acute abscess): inflammatory symptoms are more active
➢ Cold abscess (chronic abscess): inflammatory symptoms are less active
• According to etiology
➢ Infectious abscess
➢ Embolic abscess
➢ Pyemic or metastatic abscess
• Duration
➢ Acute
➢ Chronic
• According to number
➢ Single
➢ Multiple

22.Questions
a. Briefly trace iodine in food to iodine in thyroid hormone
Answer:
➢ Around 50mg of iodine in the form of iodide is required per year (1mg per
week). Source of the iodine is iodated salt. Iodine can also be found bound
to protein as iodine this has to be converted to iodide (by glutathione) as
this is the easiest form to absorbed
➢ Iodides ingested orally are absorbed from the intestine into the blood in
about the same manner as chlorides.
o Absorption of sodium ions through the epithelium creates
electronegativity in the chime (lumen) and electropositivity in the
paracellular spaces between the epithelial cells.
o Iodide ions then move along this electrical gradient to “follow” the
sodium ions.
➢ Iodide in the blood stream is transported to the thyroid glandular cells and
follicles.
➢ Iodine trapping: On the basolateral side of the glandular cells sodium-
iodide symporter co-transports 1 iodide ion along with 2 sodium ions
PREPARED BY MOSES KAZEVU JR

across the basolateral membrane into the cells. (energy to transport these
ions against a concentration gradient comes from the sodium potassium
ATPase pump).
➢ Oxidation: In the follicular cells peroxidase converts iodide ions to an
oxidized form of iodine.
➢ Organification/iodination: Iodine is transported to the colloid through
pendrin (an antiporter for chloride and iodine), in the colloid iodine is then
organified (also catalyzed by the enzyme peroxidase) to thyroglobulin
where it binds tyrosine residues to form monoiodothyronine (MIT) and
diiodothyronine (DIT).
➢ Coupling: MIT and DIT which can couple to form the hormones thyroxine
(T4) and triiodothyronine (T3) and reverse T3- also in the presence of
peroxidase enzyme
o MIT + DIT= T3
o DIT + DIT= T4
o DIT + MIT= Reverse T3
➢ The thyroid hormones can then be released in blood via pinocytosis and
secretion.
➢ More T4 is created than T3 but in the periphery 5’deiodinase converts T4
to the active T3.
PREPARED BY MOSES KAZEVU JR

b. Draw homeostatic loop for T3 and T4


Answer:

c. List signs/ symptoms of thyrotoxicosis


Answer:
➢ Symptoms
o Nervousness
o Restlessness, Anxiety and hyperactivity
o Increased sweating especially palms
o Hyperthermia and Heat intolerance
o Palpitations
o Hyper-defecation
➢ Signs
o Fine hand tremors
o Warm moist hands
o Tachycardia and atrial arrhythmias
o Systolic hypertension with wide pulse pressure
o Lid lag
o Wide eyed stare gaze
o Muscle weakness

23.Write short notes on


PREPARED BY MOSES KAZEVU JR

a. Colles’s fracture
Answer:
➢ This is a transverse fracture of the radius about 2.5cm above the wrist joint
with the following displacement (Remember mnemonic “College Rotating
Doors Shut I’m Late”):
o Rotation
o Dorsal tilt (proximal tilt)
o Dorsal shift
o Supination
o Impaction
o Lateral shift (radial shift)
o Lateral tilt
➢ It is found in elderly due to falling on an outstretched hand.
➢ Clinical features: dinner fork deformity with prominence on the back of
the wrist and depression in front, local tenderness and pain on wrist
movement
➢ Treatment:
o Closed manipulation under anesthesia
▪ Correct the impaction first
▪ Center the fragments (correct the radial displacement)
▪ Correct the proximal tilt by dorsiflexing the wrist
▪ Lock the arm towards the little finger using a slab (“corrected
dinner fork appearance”)
o X-ray: after reduction (to check of fracture is reduced-post-reduction
X-ray), after 24 hours (for neurovascular state), after 7 (to rule out any
displacement, reduce in theatre if displaced), after 14 days and at 8
weeks

b. Colostomy
Answer:
➢ Definition: this is hollow iatrogenic mucocutaneous fistula between the
colon and the skin.
➢ Types:
o Temporary: done in conditions where diversion is required to facilitate
healing distally in the rectum or distal colon. This type is closed once
purpose is over. Sites of temporary colostomy include the right
hypochondrium and left iliac fossa.
PREPARED BY MOSES KAZEVU JR

▪ Examples: Loop colostomy, Devine’s double barrel, spectacle,


Hartmann
o Permanent: this is always an end colostomy. It is placed in left iliac
fossa, 6cm above and medial to the anterior superior iliac spine.
➢ Colostomy indications:
o Congenital (congenital megacolon, anorectal malformations,
imperforate anus, Hirschsprung disease, anal atresia, anorectal
agenesis, anal stenosis) and
o Acquired (Intestinal obstruction i.e. sigmoid volvulus, ischemic bowel
disease, tumors i.e. anal and rectal carcinoma, perforation of left sided
colon, high anal fistula, after Hartmann’s operation)
➢ Complications:
o Ischemic gangrene
o Hemorrhage
o Retraction
o Prolapse
o Parastomal hernia

c. Shoulder dislocation:
Answer:
➢ This is displacement of the head of the humerus from the glenoid cavity.
➢ It can be caused by
o High speed trauma (traumatic shoulder dislocation)
o Ligament overstrain (repetitive shoulder ligament overstrain e.g. with
swimming, tennis, throwing sports e.g. baseball)
➢ Dislocation can either be partial (sublaxation) where the head of the
humerus is partially dislodged from the glenoid cavity or complete
(luxation) where the head of the humerus is completely dislodged from the
glenoid cavity.
➢ Dislocation can either be in the anterior (common) or posterior direction.
➢ Signs and symptoms:
o Deformity
o Swelling
o Numbness
o Weakness
o Bruising
o Muscle spasms
PREPARED BY MOSES KAZEVU JR

o History of trauma or sporting activity before injury


➢ On X-ray
o Loss of parallelism between the head of the humerus and the glenoid
labrum
o Ice-cream cone appearance or light bulb appearance with posterior
shoulder dislocation
➢ Treatment: closed reduction
➢ Rehabilitation: immobilize in a sling for a few weeks, apply ice to shoulder
3 to 4 times.

24.Write short notes on metabolic body response to acute blood loss.


Answer:
➢ There are 2 phases of the response
- Initial phase (EBB phase)
o Occurs within the first 48hours to 2 days
o Physiological role of the ERB phase is to conserve circulating
volume and energy stores.
o It is characterized by hypovolemia, decreased cardiac output,
decreased metabolic rate, hypothermia and lactic acidosis.
o Blood loss triggers the hypothalamus, pituitary glands, adrenals,
thyroid gland, pancreas and renin angiotensin aldosterone axis.
This results in release stress hormones (catecholamines, cortisol,
glucagon, growth hormone) and other hormones such as
aldosterone, angiotensin and thyroid hormones.
o Catecholamines produce the following changes: increase in heart
rate, respiratory rate, and temperature
▪ Accentuated clinical signs e.g. tachycardia (more than
90b/m), tachypnea (more than 20b/min), hyperthermia (38
PREPARED BY MOSES KAZEVU JR

degrees or more), hypothermia (36 degrees or less) are


indicative of systemic inflammatory response (SIRS)
▪ Any 2 of the 4 parameters above= SIRS
o Cortisol increases the blood sugar (through gluconeogenesis,
glycogenolysis)
o Aldosterone increases sodium and water reabsorption (under the
control of ADH)
o WBC count also increases and there is also increase in the
production of pro-inflammatory cytokine such as IL-1 and TNF-
alpha
- Flow phase
o Occurs from day 3 to about day 10.
o Associated with chronic neuroendocrine response.
o It is divided into:
▪ Catabolic phase day 3 to 10
▪ Anabolic phase: can last weeks depending on severity
o It is associated with the mobilization of body energy stores for
repair and replacement of damage tissue

25.Write short notes on following


A. Define sentinel lymph node
Answer: This is the hypothetical first lymph node or group of nodes draining
a cancer. In cases of established cancerous dissemination, it is postulated that
the sentinel lymph node(s) is/are the target organs primarily reached by
metastasizing cancer cells from the tumor

B. Discuss briefly value of sentinel axillary lymph node biopsy in management


of breast cancer
Answer:
The sentinel node is used during a sentinel lymph node biopsy. The procedure
consists of injecting a vital blue dye or radioactive sulfur colloid or both around
the tumor. The surgeon often waits 5 minutes in the case of the dyes and a few
hours in the case of colloid and then dissects the axilla removing only a few nodes
that either stained blue or radioactive. The principle behind this is that the first
node that drains the dye are most likely to first drain the tumor as it metastasizes.
In doing so the nodes are examined and if found to have signs of malignancy it
indicates that the breast cancer may have metastasized from the local area
PREPARED BY MOSES KAZEVU JR

however if the nodes show no signs of malignancy it is assumed that the tumor
has not metastasize. This has helped reduce morbidity as compared to a full
axillary node dissection and in experienced hands the procedure is close to 100%
accurate in correctly describing the presence or absence of tumor in the axilla.

26.Write short notes on the complications of blood transfusion


Answer:
➢ Acute hemolytic reactions due to ABO incompatibility (results from
antibodies in recipient’s plasma direct against antigens on donor’s
erythrocytes). It can be either
o Intravascular hemolysis (RBCs are hemolyzed in the blood steam) it
causes hemoglobinemia, hemoglobinuria, acute renal failure, DIC and
complement-mediated cardiovascular collapse. Dyspnea, chest pain,
sweating, fever with chills, tachycardia, hypotension and cardiac arrest
occur in fatal type. Jaundice may also be seen.
o Extravascular hemolysis: Due to Rh or non-ABO antigens. It is seen in
patients usually exposed to antigen through prior pregnancies,
transplantation or transfusion. Antibody titers are too low often to be
detected during screening however production increases with re-exposure
to the initial antigen. Antibodies do not activate complement (no
intravascular hemolysis) instead they coat RBCs which are removed by
splenic macrophages.
➢ Non-hemolytic Febrile reactions: due to formation of cytokines during
storage of blood. Most common complication due to impurities like
pyrogens in blood or in infusion set. Presents as headaches, fever, chills and
rigors, tachycardia, nausea are other features. Transfusion is temporarily
stopped or slowed down with administration of antipyretic.
➢ Allergic/anaphylactic reaction: urticaria and allergy to specific proteins in
donor’s plasma can occur.
➢ Congestive cardiac failure: occurs especially if large quantities of whole
blood are transfused in chronic severe anemia, pregnancy, elderly patients,
in patients who have cardiac problems.
➢ Infections: serum hepatitis, HIV infection, malaria infection, CMV
infection, EBV infection, Syphilis infection, Trypanosoma cruzi infection
➢ Air embolism
➢ Thrombophlebitis
PREPARED BY MOSES KAZEVU JR

➢ Hemochormatosis/iron overload in patients with chronic renal failure


receiving repeated blood transfusion
➢ Citrate intoxication causing bradycardia and hypocalcemia. (for every 4
units of blood 10ml of 10% calcium chloride or gluconate should be infused
intravenously)
➢ Transfusion related acute lung injury
➢ Graft vs Host disease: occurs when donor lymphocytes mount immune
response against the recipient lymphoid tissue.

27.What are signs and symptoms of acute appendicitis. Explain the anatomical basis
for these features, where applicable.
Answer:
➢ Symptoms:
o Pain: visceral pain starts around the umbilicus due to distension of the
appendix, after a few hours somatic pain occurs in right iliac fossa
(Blumberg sign) due to irritation of the parietal peritoneal because of
inflamed appendix. The pain is severe and diffuse. Pain migrates to right
iliac fossa with maximum tenderness at McBurney’s point.
o Vomiting and nausea due to reflex pylorospasm
o Constipation (in postileal appendix)/diarrhea (pelvic appendix)
o Anorexia and Low-grade fever
o Urinary frequency: bladder irritation from inflamed appendix
➢ Signs
o Tachycardia
o Tenderness and rebound tenderness in right iliac fossa (Blumberg
sign/release sign)
o Pointing sign- Pain at Mc Burney’s point
o Signs indicative of peritoneal irritation
▪ Rovsing sign- on pressing left iliac fossa, pain occurs in right iliac
fossa due to shift of bowel loops which irritate the peritoneum
▪ Psoas test- for retrocecal appendix, hyperextension of hip causes pain
in the right iliac fossa.
▪ Obturator test- For pelvic appendix internal rotation of right hip
causes pain in right iliac fossa due to irritation of obturator internus
muscle
PREPARED BY MOSES KAZEVU JR

▪ Baldwing test- positive in retrocecal appendix- when legs are lifted


off the bed with knees extended the patient complains of pain while
pressing on abdomen (Ribs-ilium)

28.You are called to a surgical ward, 2 hours after thyroidectomy of a patient with
thyrotoxicosis. Write short notes on what complications your patient might have
now and in the near future.
Answer:
➢ Early
- Bleeding and hematoma as a result of poor hemostasis (can lead to
upper airway obstruction
- Injury to recurrent laryngeal nerve (unilateral or bilateral)- leads to
vocal cord collapse and upper airway obstruction
- Laryngeal edema
- Tracheomalacia with large goiter that erodes the trachea that collapses
after thyroidectomy
- Upper airways obstruction
- Thyroid storm- atrial fibrillation
- Tracheal perforation
PREPARED BY MOSES KAZEVU JR

- Hypocalcemia with removal of all parathyroid glands


➢ Intermediate
- Infections
- DVT
- Hypostatic pneumonia
➢ Late
- Hypothyroidism
- Hypocalcemia with pathological fractures
- Keloids and hypertrophic scars

29.How would you investigate the cause of fever in a patient who develops a fever
3 or 4 days after abdominal surgery?
Answer:
➢ Urinalysis: Leukocyte esterase, WBCs, RBCs
➢ Urine microscopy, culture and sensitivity
➢ Blood investigations
o Full blood count with differential- increase in WBC
o ESR and C-reactive protein: increase
o Blood culture: for sepsis
o Urea and electrolytes, creatinine: to check renal function
o Liver enzymes
➢ Pus swab- microscopy, culture and sensitivity (for any pus)
➢ Imaging
o Chest X-ray (to rule out pulmonary sources of infection)
o Abdominal ultrasound
o CT scan
Assessing post-operative fever
Category Day Description
Wind 1-2 Lungs: pneumonia, aspiration
and pulmonary embolism
Atelectasis
Water 3-5 Urinary tract infection (catheter
associated)
Wound 5-7 Infection of wound either
superficial or deep
(W)abscess 5-7 Infection of an organ or space
PREPARED BY MOSES KAZEVU JR

Walking (or Veins After 5 days DVT or pulmonary embolism


pronounced “Weins”)
Wonder drugs or “What Anytime Drug fever
did we do?” Reaction to blood products
(febrile non-hemolytic or
transfusion related acute lung
injury)
Wing/waterway Anytime Bloodstream infection,
phlebitis, cellulitis related to IV
lines either central or peripheral

30.Graves disease is fairly common condition in young women in Zambia. Write


short notes on eye signs and give reasons why these signs do occur.
Answer:
➢ Exophthalmos: this is proptosis of the eye, it is caused by infiltration of
retrobulbar tissue with fluids and round cells with lid spasm of upper eyelid.
In Grave’s disease it is believed that the fibroblast have TSH receptors to
TSH like antibodies, in Grave’s disease fibroblast produces mxyoid
substances such as glycosaminoglycans which deposit in the retro-orbital
tissues.
➢ Lid retraction: the upper eyelid is higher than normal and lower eyelid is in
normal position with visible sclera adjacent to upper eyelid. It is due to
sympathetic over activity causing involuntary spasms of the smooth muscles
of the levator palpebrae superioris.
➢ Staring gaze: this is due to the absence of normal blinking. First sign to
appear. It is due to widening of palpebral fissure due to lid retraction and
also due to contraction of voluntary part of the levator palpebrae superiors
muscle.
➢ Lid lag (Von Graefe’s sign): this is inability of the upper eyelid to keep pace
with the eyeball when it looks downwards to follow the examiners finger
PREPARED BY MOSES KAZEVU JR

31.List the possible causes of PUD. Clearly explain how helicobacter pylori causes
this disease
Answer:
➢ Causes of PUD
o Infectious causes:
- Helicobacter pylori
o Non-infectious causes:
- Long term use of drugs e.g. Non-steroid anti-inflammatory drugs,
steroids
- Alcohol and smoking
- Stress
- Burns (Curling’s ulcer)
- Cerebral trauma/head injury (Cushing’s ulcer)
➢ Helicobacter pylori and peptic ulcer disease
o The organism has the enzyme urease which has the ability to hydrolyze
urea to form ammonia, a strong alkali. Ammonia through negative
feedback causes the release of gastrin from the antral G cells which results
in gastric acid hypersecretion.
o The bacterium produces enzymes e.g. dehydrogenase (converts alcohol to
aldehyde which is toxic to mucosa), and endopeptidase which disrupts the
gastric mucous barrier and the inflammation induce in the gastric
epithelium is associated with the ultimate ulceration.
o Some strains produce cytotoxins such as cagA and vacA gene products
which directly damage the cells of the gastrointestinal tract. The effect of
the organism on the gastric epithelium is to incite a classic inflammatory
response that involves the migration and degranulation of acute
inflammatory cells such as neutrophils and also the accumulation of
chronic inflammatory cells such as macrophages and lymphocytes.
PREPARED BY MOSES KAZEVU JR

32.What is gangrene? What are the etiological factors of the different types of
gangrene? Your short notes should include pathophysiology of each type of
gangrene.
Answer:
➢ Gangrene is macroscopic death of tissue in situ (in continuity with adjacent
viable tissue) with or without putrefaction. The term necrosis may be used
synonymously. It often affects the distal part of a limb because of arterial
obstruction (from thrombosis, embolus or arteritis).
o A gangrenous part lacks arterial pulsation, venous return, capillary
response to pressure, sensation, warmth and function.
o The color of the part changes through a variety of shades according to
circumstances (pallor, dusky grey, mottled, purple) until finally taking on
a the characteristic dark-brown, greenish-black or black appearance, which
is caused by the disintegration of hemoglobin and the formation of iron
sulphide.
➢ Causes
o Arterial occlusion e.g. atherosclerosis, emboli, diabetes, Raynaud’s
disease, ergots, arteritis, peripheral artery disease
o Infective: Boil, carbuncle, gas gangrene, Fournier’s gangrene, Cancrum
oris
o Traumatic: direct, indirect
o Physical: burns, scalds, frostbit, chemicals, irradiation, electrical
o Venous gangrene
o Smoking, obesity
➢ Types of gangrene
o Dry gangrene: occurs when the tissues are desiccated by gradual slowing
of the bloodstream, it is typically the result of atheromatous occlusion of
arteries. The affected part becomes dry and wrinkle, discolored from
disintegration of hemoglobin and greasy to the touch.
o Wet gangrene: occurs when infection and putrefaction are present, the
affected part becomes swollen and discolored and the epidermis may be
raised in blebs.
o Gas gangrene: It is due to infection with Clostridium species. Myonecrosis
is due to alpha-toxin from Clostridium perfringens.
o Internal gangrene: This affects one or more of the internal organs such as
the intestines, gallbladder or appendix. It occurs when blood flow to an
internal organ is blocked e.g. in appendicitis
PREPARED BY MOSES KAZEVU JR

o Fournier’s gangrene (necrotizing fasciitis) this is a rare and fulminant form


of necrotizing fasciitis of the male perineum and genital region frequently
due to a synergistic polymicrobial infection Bacteria enters through break
in the skin or it may follow surgery, trauma, IV injection or insect bite.
Infection spreads rapidly across fascial layer, leading to tissue death of
fascia and subcutaneous tissue.
o Progressive bacterial synergistic gangrene (Meleney’s gangrene): this is a
rare type of gangrene typically occurring after an operation with painful
skin lesions developing one or two weeks after surgery. It is a spreading
infectious type of gangrene and is often associated with Streptococcus
(which produces tissue damage mediated by Exotoxins A, B and C) as well
as other microbes.

33.A 26-year-old man has been hit by a moving vehicle. He is brought to casualty
in a coma. How would you evaluate this patient?
Answer:
➢ Shout for help as this requires a team effort
➢ Perform primary survey (Perform ABCs)
o A- ensure airway is patent with cervical spine protection
- Check for any signs of airway obstruction, foreign bodies, facial,
mandibular or laryngeal fractures?
- Suction any secretion
- Maneuvers such as chin lift or jaw thrust can be done to establish a
clear airway but protect the cervical spine at all times
- If airway not secure assess for the need of intubation or performing a
cricothyroidotomy or emergency tracheostomy
- If patient can talk, the airway is likely to be safe.
o B- breathing
- Evaluate breathing: perform a chest examination with adequate
exposure. Watch for chest movements (note for any paradoxical
movements in the case of flail chest that may need mechanical
ventilation) check respiratory rate, palpate for any fractures or
subcutaneous emphysema (which may require draining by needle
thoracostomy with subsequent intercostal drainage until defect is
repaired), percuss for any fluids in pleural cavity such as hemothorax
that may require intercostal drainage. Auscultate to ensure there is air
entry in the lungs.
PREPARED BY MOSES KAZEVU JR

- Provide high flow oxygen through a rebreather mask if not intubated


and ventilated.
o C- Circulation with hemorrhage control
- Assess blood loss rapidly: observe level of consciousness, skin color,
pulse, capillary refill time, bleeding
- Gain venous access with 2 large bore cannulae, obtain blood for
crossmatch and baseline investigations.
- Start running fluids, Ringer’s lactate/ normal saline rapidly as you
await blood
- Arrest any bleeding (Direct manual pressure should be used to stem
visible bleeding), perform chest, abdominal and pelvic compression to
rule out any fractures that may cause internal hemorrhage especially
when not responding to fluid resuscitation.
- Check for blood sugar
- Catheterize the patient and monitor urine output
o D- disability (neurological status)
- Assess level of consciousness, using Glasgow coma scale
- Pupil size symmetry and reaction to light
- Assess level of any spinal cord injury (limb movements, spontaneous
respiratory effort)
- Any lateralising signs
o E-exposure/environmental control
- Undress the patient but prevent hypothermia
- Check for any other injuries, factures, lacerations
- Monitor vitals for signs of deterioration
➢ Secondary survey
o After patient is stabilized, Take a complete history and full physical
examination
- Ask about events that lead to injury,
- AMPLE: any allergies, medication taken, past/present medical
illnesses, last meal, Events surrounding injury
o Document all measures done during primary survey
o Perform X-rays (Head& neck, Chest and pelvic-traumatic series X-rays),
ultrasounds (FAST scan) and other laboratory tests
o If at any point patient deteriorate revert back to primary survey
➢ Definitive care: depends on underlying injuries present.
PREPARED BY MOSES KAZEVU JR

34.Answer the following questions:


A. Define compartment syndrome and list major etiological factors
Answer:
Compartment syndrome is an entity that results from increased intra-
compartmental pressure within a limited space compromising the circulation
and function of tissues within that closed space. It is characterized by severe,
progressive and persistent pain that poorly responds to analgesics and is
aggravated by passive muscle stretch as well as paresthesia, paralysis,
pulselessness, pallor and poikilothermia.

Major etiological factors include:


o Trauma:
Fractures of the lower limb or upper limbs,
Soft tissue crush injuries
Vascular injuries
Circumferential burns affecting a compartment,
High velocity injuries: gun injuries
Snake bikes
o Iatrogenic: application of tight dressings/plasters
o Lying on one limb in comatous patients

B. Discuss briefly the pathophysiology of compartment syndrome


Answer:
Edema that is often a sequela of trauma occurring in a limited anatomical
space raises the tissue intra-compartmental hydrostatic pressure. As tissue
pressure rises, the pressure on and within the local venous system also rises.
Arterial resistance subsequently increases and inflow is reduced. If untreated
there is vascular compromise and the enclosed muscle becomes ischemic.
Prolonged ischemia results in necrosis and ultimately, fibrosis of the muscle
and surrounding soft-tissue structures. As pressure increases there is loss of
sensation in the distribution of the involved nerves and ultimately the local
pulses are lost (though sometimes the pulse may be present).
PREPARED BY MOSES KAZEVU JR

C. With aid of labelled diagram, explain Whitesides saline/needle technique for


measuring intracompartment pressure in a limb
Answer:
Intracompartment pressure measured using the Whitesides saline/needle
technique uses an 18G needle connected via a 3 way stop cock to an air filled
20ml syringe. Air filled tubing is connected to a mercury manometer and a
small amount of saline sits in the tube connected to the needle. Compression
of the syringe raises the pressure till saline flows into the compartment (this
is indicated by meniscus moving). Below is a diagram illustrating this
PREPARED BY MOSES KAZEVU JR

35.List baseline investigations and their significance in preparation of a patient due


for thyroidectomy
Answer:
• Blood investigations:
➢ Serum T3, T4 and TSH: to ensure patient is euthyroid before
thyroidectomy
➢ Baseline: urea, electrolytes, creatinine, liver function, full blood count
tests prior to thyroidectomy to check if patient is fit for surgery
➢ Blood cross match + group and save in anticipated hemorrhage during
thyroidectomy
• Imaging
➢ Neck X-ray: AP view to check for tracheal deviation, gland
calcification-bleeds easily
➢ Chest X-ray: may show tracheal deviation, lung metastasis and
retrosternal shadow
➢ Ultrasound of the thyroid to check whether it is cystic or solid
➢ ECG and ECHO of the heart- arrhythmias
➢ Radioisotope I123 scan: shows either cold, hot or warm nodules
➢ CT scan/MRI: for the anatomy and extent of the goiter as well as
relation to surrounding structures
• Fine needle aspiration cytology: shows benign or malignant cells
• Indirect laryngoscopy: to check for vocal cords (recurrent laryngeal)

36.Write short notes about Steel’s rule application in a patient with cervical spine
injury
Answer:
• The stability of the atlantoaxial articulation depends fundamentally on the
integrity of the odontoid process and ligaments. Ligament stability mostly is
maintained by 2 ligaments: transverse ligament and the alar, apical ligaments.
Failure of the transverse ligament can result in anterior translation of the atlas
on the axis.
• The anteroposterior diameter of the ring of the alas is approximately 3cm. The
spinal cord and odontoid process are each approximately 1cm in diameter,
approximately 1/3 the diameter of the ring.
• Steel’s rule of thirds states that the area inside the atlas is equally occupied in
thirds by the dens, spinal cord and space.
PREPARED BY MOSES KAZEVU JR

• The space is primarily occupied by cerebrospinal fluid allowing for some


degree of pathologic displacement. This accounts for the low incidence of
spinal cord injuries associated with C1 and C2 fractures.
• The empty space serves as a safe zone in which displacement can occur
without neurological impingement. In the presence of atlantoaxial instability,
the safe zone may decrease resulting in spinal cord compression.

37.Write shore notes on presentation and diagnosis of cervical injury


Answer:
• The most common injuries are in the regions C2, C5, C6 and C7
• The cervical spinal column is divided into anterior and posterior columns. The
anterior column contains the vertebral bodies and the posterior column
contains the spinal cord and spinous processes. An injury is considered
unstable when the injury affects both the anterior and posterior columns.
• Wedge fractures are as a result of flexion. Burst fractures are as a result of
vertical compression. Laminar fracture can either be vertical or horizontal and
are usually associated with another type of fracture.
• Presentation:
➢ Occipital-cervical spine injuries: from the occiput to C2
➢ Sub-axial cervical spine injurie: from C3 to C7
➢ Neck stiffness or pain
➢ Weakness/paralysis of innervated muscles
➢ Loss of sphincter control
➢ Decreased or absent reflexes
➢ Loss of sensation or proprioception
➢ If C3-C5 are involved abnormal breathing or respiratory failure can
occur
• Diagnosis
➢ X-ray of cervical spine: Anteroposterior (AP), lateral, oblique and
odontoid view. An adequate
➢ CT scan
PREPARED BY MOSES KAZEVU JR

38.List the various indications for doing a colostomy


Answer:
• Congenital
➢ Congenital megacolon
➢ Anorectal malformations
➢ Imperforate anus
➢ Hirschprung disease
➢ Anal atresia
➢ Anorectal agenesis
➢ Anal stenosis
• Acquired
➢ Intestinal obstruction e.g. sigmoid volvulus
➢ Ischemic bowel disease/gangrene due to strangulation
➢ Penetrating trauma to the bowel (left sided)
➢ Tumors: anal Ca and rectal Ca
➢ Perforation of left sided colon
➢ High anal fistula
➢ After Hartmann’s operation

39.Discuss in brief the causes of surgical jaundice


Answer:
• Causes may be classified as intraluminal (within the lumen), mural (within
the wall) and extramural (outside the wall) of the biliary tree
• Intraluminal
➢ Common bile duct stones
➢ Biliary stricture
➢ Pancreatitic stricture (due to chronic pancreatitis)
➢ Parasitic infestations
➢ Biliary atresia
➢ Klatskin tumor
• Mural:
➢ Cholangiocarcinoma
➢ Ascending cholangitis
➢ Sclerosing cholangitis
➢ Choledochal cysts
• Extramural
➢ Carcinoma of head and periampullary region of the pancreas
PREPARED BY MOSES KAZEVU JR

➢ Extrinsic compression of common bile duct by lymph nodes or tumors

40.Persistent generalized lymphadenopathy is a common presentation of patients to


a surgical clinical in Zambia. How would you evaluate such patients to come up
with a diagnosis? Include in your answer the possible histological diagnostic
description of the common causes.
Answer:
➢ History:
o Cough? Productive or non-productive? If production, color and amount
and is blood present?
o Night sweats
o Fatigue
o Weight loss
o Anorexia
o Fever
o Tested for HIV
➢ Physical examination
o Generalized lymphadenopathy
o High temperature, tonsils erythematous with a creamy grey exudate and
appear almost confluent usually symmetrical, hepatosplenomegaly
(infectious mononucleosis)
o Oral thrush oral hairy leucoplakia on tongue, purple lesions on hard palate
(HIV infection)
➢ Investigations
o Lymph node biopsy
o HIV Test, CD4 count and Viral load
o Full blood count with differential count
o Peripheral smear
➢ Differential diagnosis with histological findings
o HIV infection: shows follicular hyperplasia
o Tuberculosis adenitis: show caseaous necrosis
o Lymphoma: Reed-Sternberg cells seen (giant cells with large mirror image
nuclei)
o Infectious mononucleosis: giant cells with 2 large mirror image nuclei
o Leukemia
o Hyperthyroidism
o Systemic lupus erythematosus
PREPARED BY MOSES KAZEVU JR

41.A 70-year-old man presents with acute retention in a male surgical ward. Take a
brief relevant and focused history in order to arrive at a diagnosis.
Answer:
➢ Onset of the urinary retention
➢ Duration of urinary retention
➢ Number of times urine has been passed compared to normal
➢ Any associated symptoms:
o Weight loss
o Hematuria-initial, terminal or diffuse
o Poor urinary stream
o Terminal dribbling
o Dysuria
o Urgency
o Intermittency
o Back pain
➢ Any history of urinary tract infection?
➢ Any history of urinary stones?
➢ Any history of TB?
➢ Any history of acute renal failure?
➢ Last performed Digital rectal scan, any significant findings?
➢ History of prostate cancer screening?
➢ History of any urogenital surgeries or trauma

42.Write the hemodynamic changes caused by space occupying lesions in head


injury patients.
Answer:
Alterations in intracranial pressure may produce profound changes in
cardiovascular hemodynamics as there is an association between increased
intracranial pressure (which is seen in space occupying lesion in head injury
patients) and an increase of the systemic arterial pressure i.e. the systemic
hypertensive or ‘Cushing’ response.

Increase in intracranial pressure (ICP) consequently increases the pressure in the


cerebrospinal fluid. The pressure in the cerebral spinal fluid eventually rises to
the point that it meets and gradually exceeds the mean arterial blood pressure
(MAP= 2/3 Diastolic blood pressure + 1/3 systolic blood pressure). When the
PREPARED BY MOSES KAZEVU JR

ICP exceeds the MAP, arterioles located in the brain’s cerebrum become
compressed and there is diminished blood supply to the brain (cerebral ischemia)

During the increase in ICP, both sympathetic nervous system and


parasympathetic nervous system are activated. In the first stage, sympathetic
nervous system stimulation is much greater than parasympathetic. The
sympathetic response activates alpha 1 adrenergic receptors causing constriction
of the body’s arteries this increases the system vascular resistance and blood
pressure (hypertension). This is mean to increase blood flow to the ischemic brain
as Cerebral perfusion pressure (CPP) is dependent on the mean arterial pressure
and intracranial pressure (CPP= MAP- ICP).

The sympathetic stimulation also increases the heart contractions and cardiac
output. Tachycardia may be seen in the early stages of the Cushing reflex.
Meanwhile baroreceptors in the aortic arch detect the increase in blood pressure
and trigger a parasympathetic response via the vagus nerve resulting in
bradycardia in the second stage. This increase in parasympathetic output is also
thought to be responsible for cushing’s ulcers in the stomach due to uncontrolled
activation of parietal cells.

As long as ICP is high, blood pressure will remain high in an attempt to overcome
the resisting pressure of the compressed cerebral arteries however if this is
insufficient hypoxia and infarction ensure. Furthermore, endogenous stimulation,
raised ICP and tachycardia leads to increased pressure on the brainstem and this
can cause irregular respiratory pattern and/or apnea (the third and final stage of
the reflex)

Hemodynamic changes
➢ Systolic hypertension with widening pulse pressure
➢ Bradycardia

43.Write short notes on flail chest.


Answer:
• This is a life-threatening medical condition that occurs when 2 or more
consecutive ribs are fractured in 2 or more places
• Flail chest compromises a patient’s breathing because the fractured segment
undergoes paradoxical movement during inspiration. The segment moves
PREPARED BY MOSES KAZEVU JR

inwards rather than outwards and hence impedes ventilation of the ipsilateral
lung.
• It is usually accompanied by pulmonary contusion (bruise of the lung tissue
that can interfere with blood oxygenation)
• Etiology: mostly linked to trauma
• Clinical features: chest pain and shortness of breath, and paradoxical
respiration
• Diagnosis: clinical. A plain X-ray or CT scan can be done
• Management:
➢ Analgesia
➢ Strapping of flail segment
➢ Positive pressure ventilation
➢ Chest tube may be required
➢ Physiotherapy
• Complications:
➢ Pneumothorax
➢ Respiratory failure

44.Answer the following


A. Define osteoarthritis
Answer: This is a degenerative bone and joint conditions initially involving
thinning of the hyaline cartilage until the underlying bone is exposed and
becomes damaged due to direct stress

B. Discuss briefly pathogenesis of degenerative osteoarthritis


Answer:
Osteoarthritis (OA) is traditionally thought of as a ‘wear and tear’ disease
which occurs with increase in age. It may be primary (idiopathic) or secondary
to an underlying pathology. The pathogenesis of OA involves a degradation
of cartilage and remodeling of bone due to an active response of chondrocytes
in the articular cartilage and the inflammatory cells in the surrounding tissue.
There is initial thinning of the hyaline cartilage until the underlying bone is
exposed and becomes damaged due to direct stress.

The release of enzymes from these cells break down collagen and
proteoglycan destroying the articular cartilage. The exposure of the
PREPARED BY MOSES KAZEVU JR

underlying subchondral bone results in sclerosis followed by reactive


remodeling changes that lead to formation of osteophytes and subchondral
bone cysts. The joint space is progressively lost over time.

C. List the four cardinal radiological features of osteoarthritis


Answer:
➢ Loss of joint space (asymmetrical)
➢ Osteophytes/ bone lipping
➢ Subchondral bone sclerosis
➢ Subchondral bone cysts
45.Write short notes on septic shock
46.Significance of monitoring blood pressure and pulse rate in a head injury patient
47.Write short notes on clinical presentation of inguinal hernia
48.Write short notes on eye signs in thyrotoxicosis
49.Discuss briefly the common traction methods, their uses and complications
50.Write short notes on a patient who sustained fracture-dislocation at levels
CV5/CV6
51.Significance of finding unequal pupils in a head injury patient
52.Significance of baseline blood pressure, pulse rate, and respiratory rate of a poly
trauma patient
53.Describe briefly approach to hemolytic blood transfusion reaction. List causes,
signs and symptoms and treatment of
A. Hyperkalemia
B. Hypocalcemia
C. Hypernatremia
54.Write short notes on clinical presentation and investigations in rectal carcinoma
55.Acute osteomyelitis
56.A fifty year old woman comes with a breast lump. Write short notes on how you
would assess this patient to arrive at a possible diagnosis.
PREPARED BY MOSES KAZEVU JR

57.A 65-year-old patient presents with a painful ulcer on the dorsum of his foot. He
complains that he is unable to sleep at night and the pain is not relieved by
moderate pain killer
A. List 2 differential diagnosis for this ulcer
Answer:
➢ Ischemic ulcers (arterial ulcer)
➢ Venous ulcer

B. What is the most likely diagnosis?


Answer: Ischemic ulcer

C. What 4 important things would you ask in the history to determine the
etiologic factors?
Answer:
➢ Medical history of diabetes and hypertension
➢ History of stroke, myocardial infarction
➢ Intermittent claudication
➢ Diet/Cholesterol levels

D. What 2 blood investigations are indicated?


Answer:
➢ Random blood sugar
➢ Cholesterol levels

E. What radiological investigation is indicated?


Answer: Doppler ultrasound

F. Describe the following terms: a. Ankle Brachial pulse index and b. critical
ischemia?
Answer
➢ Ankle brachial pulse index is the ratio of brachial pressure over that of the
ankle pressure. It is usually 1
➢ Critical ischemia is present when this ratio is less than 0.6
PREPARED BY MOSES KAZEVU JR

49.A 47-year-old female patient is admitted to casualty with swelling of her left leg
of spontaneous onset. She has a fever and the leg feels warm
A. List 2 differentials diagnosis
Answer:
Cellulitis
Deep vein thrombosis

B. What bacteria is likely to be the cause?


Answer: Streptococcus pyogenes

C. What 2 important questions would you ask for in the history to help determine
the etiological factors?
Answer:
➢ Past medical history of diabetes/immunosuppression
➢ History of trauma/injury

D. What 4 signs would you look for in the examination?


Answer:
➢ Swelling
➢ Hotness
➢ Redness
➢ Pain

E. What 2 blood investigations would you do?


Answer:
➢ Full blood count with differential count
➢ Blood culture/C-reactive protein

F. What antibiotic is indicated and what route of administration is


recommended?
Answer: Penicillins intravenous
PREPARED BY MOSES KAZEVU JR

50.A 15-year-old boy presents to the casualty department with scrotal pain. He gives
a history of sudden onset with no history of trauma. Answer the questions below.
A. What are the 2 main differentials?
Answer:
Testicular torsion
Epididymorchitis

B. What 4 things would you ask for in the history to differentiate the two?
Answer:
➢ History of nausea and vomiting
➢ Previous episode
➢ Precipitating factors e.g. sex, masturbation, pornographic materials
➢ History of urethritis

C. What is the most likely diagnosis?


Answer: Testicular torsion

D. What is Prehn’s test? Describe how it is done.


Answer: Prehn’s test is done by elevation of the testis. In testicular torsion the
pain increase whilst in epididymitis it reliefs the pain

E. What radiological investigation is indicated?


Answer: Color doppler ultrasound

51.A 12-year-old boy presents to clinic 3 with a swelling on the upper leg which is
fast growing. The child is ill and give history of weight loss
A. List 2 differential diagnosis
Answer:
Osteosarcoma
Osteomyelitis

B. What radiological investigation might distinguish the two?


Answer: Plain X-ray

C. What other radiological investigation can be done?


Answer: Technetium bone scan
PREPARED BY MOSES KAZEVU JR

D. List 2 X-ray findings in osteosarcoma


Answer:
Codman triangle
Sunray appearance/spicules

52.A game ranger is brought to the casualty with a gunshot wound to the right leg.
The wound is dirty and 5cm in length. The patient has palpable pulses and nerve
function is normal.
A. What type of fracture has the patient had?
Answer: Open tibia fracture

B. What is the classification of open tibia fractures?


Answer:
This is the Gustillo-Anderson classification. It consists of 3 types
Type 1: Wounds are <1cm and clean (mild contamination), simple fracture
pattern and there is no skin crushing
Type 2: Wounds measuring 1-10 cm but without significant soft tissue
crushing, flaps, degloving or contusion. Fracture pattern is more complex and
there is moderate contamination
Type 3: Wounds >10cm, also includes injuries older than 8 hours. There is
severe contamination. It has 3 subtypes
o Type 3A: no periosteal stripping
o Type 3B: periosteal stripping
o Type 3C: neurovascular damage

C. How would you classify this patient’s fracture?


Answer: Type 3B

D. What blood investigations would you do?


Answer:
➢ Full blood count with differential white cell count
➢ Blood cultures

E. What 2 organisms would be present on pus swab of this wound?


Answer:
➢ Anaerobic Clostridium tetani
➢ Bacteroides
PREPARED BY MOSES KAZEVU JR

58.A 46-year-old Zambian female presents to clinic 4 with a thyroid swelling which
is painless and slow growing.
A. What 4 things would you ask in the history to determine the cause?
Answer:
➢ Where she lives (mountainous or valley) e.g. Western province
➢ Diet (goitrogens, chelators etc.) e.g. cabbage
➢ Voice changes (malignancy)
➢ Eye changes (toxic symptoms): popping out, tearing, redness and
double vision
B. What 4 examinations would you do to determine whether the patient is
euthyroid or toxic?
Answer:
➢ Hand examination for fine tremors, sweaty palms
➢ Eye examination for proptosis, chemosis, ophthalmoplegia
➢ Cardiovascular examination for arrhythmias, tachycardia
➢ Leg for pretibial myxedema

C. What specific blood tests are indicated?


Answer: T3, T4, TSH and thyroid stimulating hormone antibodies

D. What 3 radiological investigations would you do?


Answer:
➢ X-ray neck (AP and lateral)
➢ Ultrasound of thyroid gland
➢ Radioisotope scan

E. List 4 indications for surgery in this patient


➢ Malignancy
➢ Failed medical treatment
➢ Cosmesis
➢ Compression symptoms (trachea and esophagus)
PREPARED BY MOSES KAZEVU JR

59.A 37-year-old female patient is seen in clinic 4 with a lump in the breast and an
axillary swelling. She gives history of weight loss and back ache. Answer the
questions below.
A. List 2 differential diagnosis
Answer:
➢ Carcinoma of the breast
➢ Phylloides tumor

B. What is the most likely diagnosis?


Answer: carcinoma of the breast

C. What 4 important questions would you ask to determine the possible


etiological factors?
Answer:
➢ Family history of Ca breast
➢ Onset of menarche
➢ Contraceptive use
➢ Nulliparity
➢ Others: late first pregnancy, breast feeding etc.

D. List 2 investigations you would do to confirm your diagnosis


Answer:
➢ Fine needle aspiration cytology
➢ Mammography

E. What 2 radiological investigations would you do to determine whether


metastasis is present?
Answer:
➢ Bone scan
➢ Chest Xray
➢ Others: Ultrasound of the liver CAT brain

F. What is the value of the sentinel node in this condition?


Answer: to determine axillary spread of tumor
PREPARED BY MOSES KAZEVU JR

60.A 34-year-old woman presents with mild yellowness of eyes, pain in the right
hypochondrium and fever. She gives history of previous episodes which resolve
on antibiotic treatment. Answer the questions below
A. What is the most likely diagnosis?
Answer: Acute cholecystitis

B. What is the most likely causes?


Answer: Gall stones (Cholelithiasis)

C. What other 2 things would you ask for in the history?


Answer:
Pain with fatty meals
Pain radiating to right scapula area

D. What 2 blood investigations would you ask for?


Answer
Bilirubin levels (liver function tests)
Alkaline phosphatase
Others: serum amylase

E. What 2 signs would be present on abdominal examination?


Answer:
Murphy’s sign
Monihyums sign

F. What radiological investigation is indicated?


Answer: Ultrasound liver

61.A 56-year-old patient is seen in casualty with history of failing to pass urine. He
has severe pain and abdominal distention.
A. What is the diagnosis?
Answer: acute urinary retention

B. List 2 possible conditions that can lead to this complication


Answer:
➢ Benign prostatic enlargement
➢ Cancer of the prostate
PREPARED BY MOSES KAZEVU JR

C. List 4 questions you would ask to differentiate the 2 conditions


Answer:
➢ Severe backache
➢ Weight loss
➢ Age
➢ Hematospermia

D. What single physical examination is necessary in this patient?


Answer: Digital rectal examination

E. What one endoscopy and one radiological investigation should be done?


Answer:
Cystoscopy
Ultrasound kidneys, bladder and prostate

62.A 35-year-old patient is seen in casualty following a road traffic accident (RTA).
The patient smells of alcohol and has a laceration on the occipital area. Answer
the questions below briefly.
A. What two things would you ask in the history?
Answer:
➢ Loss of consciousness
➢ Amnesia (pre- or post) traumatic

B. Explain how you would examine the occipital laceration


Answer: gloved finger should palpate for any fracture in the bone through the
laceration

C. How would you assess the level of consciousness of this patient?


Answer: Glasgow coma scale chart: eye opening, verbal response and motor
response

D. List 4 localizing signs you would look for


Answer:
➢ Side on laceration is (right or left)
➢ Pupillary signs (e.g. asymmetric size)
➢ Cranial nerve palsy (asymmetrical)
➢ Hemiplegia/Hemiparesis
PREPARED BY MOSES KAZEVU JR

E. List 4 reasons you might ask for a CT scan in this patient


Answer:
Glasgow coma scale <7
History of lucid interval
High alcohol ingestion and difficult to examine
Any localizing sign

63.Discuss briefly common patterns and presentation of hand infection.


Answer:
➢ General features:
o Infection spreads faster in all areas.
o Infection causes edema over the dorsum of the hand due to lax skin
and more lymphatic network even though infection per se is more over
the volar aspect. Hand looks like a frog hand.
o There is restricted movement of fingers and hands.
o The hand functions like hook, pinch, grip, grasp are lost.
o There is severe pain and tenderness with fever.
o Tender palpable axillary lymph nodes are often present
o They can be superficial or deep and can be localized or spreading
➢ Different types of hands infection include:
o Acute paronychia (most common hand infection, occurs in
subcuticular area under the eponychium) presents as severe throbbing
pain and tenderness with visible pus under the nail root. Nail on touch
is very tender. Caused by minor injury, implicated organisms
Staphylococcus aureus and Streptococcus pyogenes
o Chronic paronychia- commonly due to fungal infection (Candida), nail
is diseased with ridges and pigmentation, itching in the nail bed,
recurrent pain, and discharge. Secondary bacterial infection may
supervene
o Terminal pulp space infection (felon)- second most common hand
infection. Affect index and thumb mostly, usually caused by minor
injury like finger prick. Presents with pain, tenderness, swelling in the
terminal phalanx, fever and tender axillary lymph nodes. Implicated
organisms include staphylococcus, streptococcus and gram-negative
organisms
PREPARED BY MOSES KAZEVU JR

o Subungual infection
o Web space infection
o Mid-palmar space infection
o Thenar space infection
o Deep palmar abscess
o Acute suppurative tenosynovitis
o Chronic tenosynovitis of flexor tendon sheath of palm and forearm-
compound palmar ganglion
o Lymphangitis of the hand
o Arthritis of hand joint
o Subcuticular abscess

64.There has been a plane crash in Ngwerere area with a few survivors. How would
you handle the 13 survivors who have been brought to casualty?

65.A 65-year-old male patient presents to the casualty with a very painful ulcer on
the right foot. He is a known hypertension and has been on treatment for several
years. On examination has a moderate sized ulcer on the dorsum of the foot which
is very painful. The ulcer has punched out edge and the base is pale. The SHO
makes a diagnosis of a venous ulcer.
A. What other history would you want to get from this patient?
B. What further clinical examination would you do?
C. Was the diagnosis correct? Give your reasons?
D. What investigation is indicated
E. What is the diagnosis and state your reasons?

66.The signs of intracranial bleeding after a head injury are


A. Conscious level ……………………………………….
B. Pupil on side of lesion ……………………………..
C. Pulse rate …………………………………………………
D. Blood pressure …………………………………………
E. Respiration ……………………………………………....
PREPARED BY MOSES KAZEVU JR

31.Give an account of thyroid swellings

32.Define the following


A. Fistula
B. Fissure
C. Ulcer
D. Granulation tissue
E. Paralytic ileus

33. A 40-year-old man comes to the emergency ward with acute urinary retention.
What are the possible etiologies, briefly outline how you would investigate.

34.A beautiful 40-year-old female patient is seen in casualty with right upper
quadrant pain which radiates to the scapula. She gives history of intermittent
attacks associated with fatty foods. On examination Murphy’s sign and
Moynihan’s sign are both positive. The SHO on call makes a diagnosis of Acute
pancreatitis and requests some blood tests. The result shows serum amylase 100
u/L, Alkaline Phosphates 300 u/L, SGPT 30u/L, SGOT 31u/L and bilirubin 25
micronmols/L. The SHO is uncertain of the interpretation of the results and calls
the medical Registrar on call. The Registrar takes the history on the phone. The
medical Registrar is extremely upset with the young SHO, but nonetheless
recommends IV antibiotics, analgesia and an urgent U/S. By the morning round
the SHO has altered his diagnosis and takes the credit for an astute diagnosis from
the newly appointed Prof of Surgery.
A. What other important history should be taken?
B. What diagnosis do both the history and examination point to?
C. What do the investigations show?
D. Is a plain X-ray indicated in this patient?
E. Why was the medical registrar so upset?
F. What were the surgical principles on which the medical registrar gave his
advise?
PREPARED BY MOSES KAZEVU JR

53.Write short notes on usefulness of intravenous urogram (IVU) in a patient with


bilateral hydronephrosis

54.Write short notes on how you would manage a patient you found to have a
ruptured liver during an operation in a district hospital.

55.Discuss the various hormones that are involved following the ruptured spleen in
a 25-year-old man.

You might also like